++
++
++
++
Deficits in the complement membrane attack complex (C5-8) are associated with infections of what variety?
++
++
++
A. Catalase-positive bacteria
++
++
B. Neisseria meningitidis
++
++
C. Pseudomonas aeruginosa
++
++
++
++
E. Streptococcus pneumoniae
+
++
++
(Chap. 1) Deficiencies in the complement system predispose patients to a variety of infections. Most of these deficits are congenital. Patients with sickle cell disease have acquired functional defects in the alternative complement pathway. They are at risk of infection from Streptococcus pneumoniae and Salmonella spp. Patients with liver disease, nephrotic syndrome, and systemic lupus erythematosus may have defects in C3. They are at particular risk for infections with Staphylococcus aureus, S. pneumoniae, Pseudomonas spp, and Proteus spp. Patients with congenital or acquired (usually systemic lupus erythematosus) deficiencies in the terminal complement cascade (C5-8) are at particular risk of infection from Neisseria spp such as Neisseria meningitidis or N. gonorrhoeae.
++
++
++
A 63-year-old man has chronic obstructive pulmonary disease and presents to your office for routine follow-up. He has no complaints currently and feels well. His most recent forced expiratory volume in 1 second (FEV1) was 55% predicted, and he is not on oxygen. He has received one dose of pneumococcal vaccine 7 years previously. He is asking if he should receive another dose of pneumococcal vaccine. According to the guidelines of the Centers for Disease Control and Prevention, what is your recommendation?
++
++
++
A. He does not require further vaccination unless his FEV1 drops below 50% predicted
++
++
B. He does not require further vaccination until he reaches age 65
++
++
C. He should be revaccinated today
++
++
D. He should be revaccinated 10 years after his initial vaccine
++
++
E. No further vaccination is recommended as a single dose is all that is required
+
++
++
(Chap. 5) Pneumococcal vaccination has been recommended for all individuals at any age with a variety of chronic medical conditions, including chronic respiratory disease, chronic heart disease, chronic liver failure, diabetes mellitus, asplenia, and chronic kidney disease. Determining when to revaccinate individuals has been somewhat controversial. The current recommendations are to revaccinate individuals’ ages 19–64 five year after the initial vaccine if they have chronic renal failure or nephrotic syndrome, asplenia, or other immunocompromising conditions. All other individuals should receive a one-time revaccination at age 65 and older if they were vaccinated 5 or more years previously and were age <65 at the time of original vaccination.
++
++
++
Which of the following immunizations is required for entry into many countries in sub-Saharan Africa?
++
++
++
++
++
++
++
++
++
++
++
+
++
++
(Chap. 6) When traveling abroad, it is important to plan ahead and consider the potential infectious agents to which one might be exposed. The Centers for Disease Control and Prevention and the World Health Organization publish guidelines for recommended vaccinations prior to travel to countries around the world. Prior to travel, it is certainly recommended that an individual be up to date on all routine vaccinations, including measles, diphtheria, and polio. Influenza is perhaps the most common preventable illness in travelers, and the influenza vaccine should be administered per routine guidelines. There are however very few required vaccinations in most countries. Yellow fever is one exception, and proof of vaccination is required by many countries in sub-Saharan Africa and equatorial South America. This is especially important for individuals traveling from areas where yellow fever is endemic or epidemic. The only other required vaccinations are meningococcal meningitis and influenza vaccination to travel in Saudi Arabia during the Hajj.
++
++
++
All of the following are reasons that troops deployed on foreign soil are at risk of acquiring infectious diseases endemic to those foreign areas EXCEPT:
++
++
++
A. Crowded social conditions engendered by mass troop deployments
++
++
B. Immunologic naiveté with regard to local endemic or enzootic pathogens
++
++
C. Lapses in hygiene and sanitation that accompany armed conflicts
++
++
D. Medical facilities that are decreased in quantity and often inadequate
++
++
E. Population displacements
+
++
++
(Chap. 9) Multiple factors contribute to infectious risks for soldiers serving overseas. The clinical spectrum of infectious illness acquired overseas includes acute infections in the combat theater, acute infections with delayed symptoms, and chronic or relapsing infections. Infectious illnesses were once a major cause of noncombat mortality but the increase use of preventative vaccines and early antimicrobial therapy has significantly decreased this mortality. Nevertheless, they remain an important source of morbidity. Typically military medical facilities are of high quality and distributed adequately to support the health care needs of troops.
++
++
++
Microbial agents have been used as bioweapons since ancient times. All of the following are key features of microbial agents that are used as bioweapons EXCEPT:
++
++
++
A. Environmental stability
++
++
B. High morbidity and mortality rates
++
++
C. Lack of rapid diagnostic capability
++
++
D. Lack of readily available antibiotic treatment
++
++
E. Lack of universally available and effective vaccine
+
++
++
(Chap. 10) Microbial agents have been used as bioweapons as far back as the 6th century B.C. when water supplies were poisoned with Claviceps purpurea by the Assyrians. In modern times, science that has been often sponsored by governmental agencies has led to new ways to enhance and spread microbial bioweapons. Bioterrorism should be delineated from biowarfare. While bioterrorism has the potential to lead to thousands of deaths if employed in a large scale manner, the primary impact is the fear and terror generated by the attack. However, biowarfare specifically targets mass casualties and seeks to weaken the enemy. The Working Group for Civilian Biodefense has outlined key features that characterize agents that are the most effective bioweapons. These ten features are:
++
High morbidity and mortality rates
Potential for person-to-person spread
Low infective dose and highly infectious by the aerosol route
Lack of rapid diagnostic capability
Lack of universally available effective vaccine
Potential to cause anxiety
Availability of pathogen and feasibility of production
Environmental stability
Database of prior research and development
Potential to be weaponized
++
A lack of effective and available treatment is not one of the characteristics of an effective bioweapon. Bacillus anthracis is the causative organism of anthrax, one of the most prototypical microbial bioweapons, but many antibiotics have efficacy against anthrax and can be life-saving if initiated early.
++
++
++
Hyperthermia is defined as:
++
++
++
A. A core temperature >40.0°C
++
++
B. A core temperature >41.5°C
++
++
C. An elevated temperature that normalizes with antipyretic therapy
++
++
D. An uncontrolled increase in body temperature despite a normal hypothalamic temperature setting
++
++
E. Temperature >40.0°C, rigidity, and autonomic dysregulation
+
++
++
(Chap. 11) Hyperthermia occurs when exogenous heat exposure or an endogenous heat-producing process, such as neuroleptic malignant syndrome or malignant hyperthermia, leads to high internal temperatures despite a normal hypothalamic temperature set point. Fever occurs when a pyrogen, such as a microbial toxin, microbe particle, or cytokine, resets the hypothalamus to a higher temperature. A particular temperature cutoff point does not define hyperthermia. Rigidity and autonomic dysregulation are characteristic of malignant hyperthermia, a subset of hyperthermia. Fever, not hyperthermia, responds to antipyretics.
++
++
++
A 23-year-old woman with a chronic lower extremity ulcer related to prior trauma presents with rash, hypotension, and fever. She has had no recent travel or outdoor exposure and is up to date on all of her vaccinations. She does not use IV drugs. On examination, the ulcer looks clean with a well-granulated base and no erythema, warmth, or pustular discharge. However, the patient does have diffuse erythema that is most prominent on her palms, conjunctiva, and oral mucosa. Other than profound hypotension and tachycardia, the remainder of the examination is nonfocal. Laboratory results are notable for a creatinine of 2.8 mg/dL, aspartate aminotransferase of 250 U/L, alanine aminotransferase of 328 U/L, total bilirubin of 3.2 mg/dL, direct bilirubin of 0.5 mg/dL, INR of 1.5, activated partial thromboplastin time of 1.6 × control, and platelets at 94,000/μL. Ferritin is 1300 μg/mL. The patient is started on broad-spectrum antibiotics after appropriate blood cultures are drawn and is resuscitated with IV fluid and vasopressors. Her blood cultures are negative at 72 hours: at this point her fingertips start to desquamate. What is the most likely diagnosis?
++
++
++
A. Juvenile rheumatoid arthritis (JRA)
++
++
++
++
C. Staphylococcal toxic shock syndrome
++
++
D. Streptococcal toxic shock syndrome
++
++
+
++
++
(Chap. 12) This case is likely toxic shock syndrome, given the clinical appearance of septic shock with no positive blood cultures. The characteristic diffuse rash, as well as the lack of a primary infected site, makes staphylococcus the more likely inciting agent. Streptococcal toxic shock usually has a prominent primary site of infection, but the diffuse rash is usually much more subtle than in this case. Staphylococcal toxic shock can be associated with immunosuppression, surgical wounds, or retained tampons. Mere Staphylococcus aureus colonization (with an appropriate toxigenic strain) can incite toxic shock—overt infection is not necessary. Centers for Disease Control and Prevention guidelines state that measles, Rocky Mountain spotted fever, and leptospirosis need to be ruled out serologically to confirm the diagnosis. However, this patient is at very low risk for these diagnoses based on vaccination and travel history. JRA would become a consideration only if the fevers were more prolonged and there was documented evidence of organomegaly and enlarged lymph nodes.
++
++
++
You run over your upcoming daily clinic patients and note that you have quite a group this morning. Which of the following patients scheduled to see you today warrants the diagnosis of fever of unknown origin (FUO)?
++
++
++
A. A 29-year-old woman with nearly daily fever of 38.3°C for 3 weeks. She last saw you a week prior and complained of a facial rash, bilateral metacarpophalangeal arthralgias, and fatigue. Laboratory work-up revealed proteinuria, anemia, and a positive test for anti-nuclear antibodies (ANA) at a titer of >1:640.
++
++
B. A 64-year-old man whom you saw 3 weeks ago. At that time, he had 1 week of twice daily fever to 39.2°C, rigors, and a sore throat. He has since recovered and is asymptomatic now, but no cause was found despite extensive testing.
++
++
C. A 33-year-old man with subjective feeling of having a fever several times per week for 1 month. He also has rigors and an evanescent rash that has appeared on his legs, torso, and back.
++
++
D. A 45-year-old man with 4 weeks of nearly daily fevers to 38.5°C and crippling shin pain. You last saw him 1 week prior, and a comprehensive work-up including erythrocyte sedimentation rate (ESR) and C-reactive protein (CRP) levels, complete blood count, electrolytes, lactate dehydrogenase, ferritin, antinuclear antibodies, rheumatoid factor, urinalysis, blood and urine cultures, chest x-ray, abdominal ultrasonography, and tuberculin skin test have all been unrevealing.
++
++
E. A 26-year-old woman on mycophenolate mofetil, tacrolimus, and prednisone after having undergone living unrelated donor kidney transplant 1 year prior. She has had fevers to >38.3°C for just over 3 weeks. Evaluation 2 weeks ago included unrevealing erythrocyte sedimentation rate (ESR) and C-reactive protein (CRP) levels, complete blood count, electrolytes, lactate dehydrogenase, ferritin, antinuclear antibodies, rheumatoid factor, urinalysis, blood and urine cultures, chest x-ray, abdominal ultrasonography, and tuberculin skin test.
+
++
++
(Chap. 13) Many clinicians incorrectly employ the term fever of unknown origin (FUO), using it to mean any fever without an initially obvious etiology. However, the term FUO connotes a very specific set of criteria, and should be reserved for prolonged febrile illnesses without an established etiology despite intensive evaluation and diagnostic testing. These are:
++
Fever >38.3°C (101°F) on at least two occasions
Illness duration of ≥3 weeks
No known immunocompromised state
Diagnosis that remains uncertain after a thorough history-taking, physical examination, and the following obligatory investigations: determination of erythrocyte sedimentation rate (ESR) and C-reactive protein (CRP) level; platelet count; leukocyte count and differential; measurement of levels of hemoglobin, electrolytes, creatinine, total protein, alkaline phosphatase, alanine aminotransferase, aspartate aminotransferase, lactate dehydrogenase, creatine kinase, ferritin, antinuclear antibodies, and rheumatoid factor; protein electrophoresis; urinalysis; blood cultures (n = 3); urine culture; chest x-ray; abdominal ultrasonography; and tuberculin skin test (TST).
++
Only the patient described in option D meets these criteria. The patient described in option A meets criteria for a diagnosis of systemic lupus erythematosus, so the etiology is not unknown for her fever. The patient described in option B is no longer having fevers, and his duration of febrile illness was not >3 weeks. The patient in option C has no objective evidence of fever, and has not undergone any of the obligatory evaluation for obvious etiologies. The patient in option E is immunocompromised. These patients’ work up requires an entirely different diagnostic and therapeutic approach, and they are not included in the diagnosis of FUO.
++
++
++
A 42-year-old man with a history of epilepsy presents to the emergency department after having a seizure. He is on maintenance levetiracetam for seizure prophylaxis at home and has been taking his medications as prescribed. In the emergency department, he has another generalized tonic clonic seizure and is postictal after the event. He is given a dose of phenobarbital and admitted to the hospital. Over the ensuing 24 hours, the above-depicted skin eruption developed which demonstrates early desquamation. He also developed hypotension requiring escalating vasopressor support, facial edema, generalized lymphadenopathy, abnormal liver function tests, and hepatomegaly. Differential on his complete blood count is below:
++
++
Which of the following is the most likely diagnosis?
++
++
++
A. Drug reaction with eosinophilia and systemic symptoms (DRESS)
++
++
B. Eosinophilic non-Hodgkin’s lymphoma with Sweet’s syndrome
++
++
++
++
D. Staphylococcus toxic shock syndrome
++
++
E. Stevens-Johnsons syndrome
+
++
++
(Chaps. 12 and 14) This patient exhibits classic findings of the drug reaction with eosinophilia and systemic symptoms syndrome (DRESS). Some individuals are genetically unable to detoxify arene oxides present in some anticonvulsants (e.g., phenobarbital) and are susceptible to this dire syndrome. The confluence of this desquamative rash, eosinophilia, hepatic involvement, facial edema, and hypotension are all typical of this disease. Sweet syndrome or acute febrile neutrophilic dermatosis is a characterized by erythematous indurated plaque with a pseudovesicular border. In 20% of cases, it is associated with malignancy (usually hematologic), but can also be associated with infections, inflammatory bowel disease, or pregnancy. Erythema multiforme is characterized by target lesions (central erythema surrounded by area of clearing and another rim of erythema) up to 2 cm, which are symmetric on the knees, elbows, palms, and soles and spread centripetally. It is often confused with Stevens-Johnson syndrome, though erythema multiforme lacks the marked skin sloughing seen in Stevens-Johnson syndrome. Staphylococcus toxic shock syndrome is a consideration here, as the hypotension and skin rash are typical. However, the lack of a cutaneous lesion or other risk factor along with the concomitant eosinophilia and hepatitis make DRESS the more likely diagnosis.
++
++
++
A 63-year-old man is treated with paclitaxel and carboplatin chemotherapy for stage IIIB adenocarcinoma of the lung.
++
He presents for evaluation of a fever to 38.3°C (100.9°F). He is found to have erythema at the exit site of his tunneled catheter although the tunnel itself is not tender or red. Blood cultures are negative at 48 hours. His neutrophil count is 1550/µL. What is the best approach to the management of this patient?
++
++
++
A. Removal of catheter alone
++
++
B. Treatment with ceftazidime and vancomycin
++
++
C. Treatment with topical antibiotics at the catheter site
++
++
D. Treatment with vancomycin alone
++
++
E. Treatment with vancomycin and removal of catheter
+
++
++
(Chap. 15) Clinicians are often faced with treatment decisions regarding catheter-related infections in patients who are immunocompromised from cancer and chemotherapy. As many patients are requiring several weeks of chemotherapy, tunneled catheters are often placed, and determining the need for catheter removal is an important consideration. When blood cultures are positive or there is evidence of infection along the track of the tunnel, catheter removal is recommended. When the erythema is limited to the exit site only, then it is not necessary to remove the catheter unless the erythema fails to respond to treatment. The recommended treatment for an exit site infection should be directed against coagulase-negative staphylococci. In the options presented, vancomycin alone is the best option for treatment. There is no need to add therapy for Gram-negative organisms as the patient does not have neutropenia and has negative cultures.
++
++
++
During the first 2 weeks following solid organ transplantation, which family of infection is most common?
++
++
++
A. Cytomegalovirus (CMV) and Epstein-Barr virus (EBV) reactivation
++
++
B. Humoral immunodeficiency-associated infections (e.g., meningococcemia, invasive Streptococcus pneumoniae infection)
++
++
C. Neutropenia-associated infection (e.g., aspergillosis, candidemia)
++
++
D. T cell deficiency-associated infections (e.g., Pneumocystis jiroveci, nocardiosis, cryptococcosis)
++
++
E. Typical hospital-acquired infections (e.g., central line infection, hospital-acquired pneumonia, urinary tract infection)
+
++
++
(Chap. 16) Ultimately, solid organ transplant patients are at highest risk for infection due to T cell immunodeficiency from antirejection medicines. As a result, they are also at risk for reactivation of many of the viruses from the Herpes virus family, most notably CMV, varicella-zoster virus, and EBV. However, immediately after transplant, these deficits have not yet developed in full. Neutropenia is not common after solid organ transplantation as in bone marrow transplantation. In fact, patients are most at risk of infections typical for all hospitalized patients, including wound infections, urinary tract infection, pneumonia, Clostridium difficile infection, and line-associated infection. Therefore, a standard evaluation of a febrile patient in the first weeks after a solid organ transplant should include a detailed physical examination, blood and urine cultures, urinalysis, chest radiography, C. difficile stool antigen/toxin studies if warranted, in addition to a transplant-specific evaluation.
++
++
++
After leaving which of the following patient’s room, would the use of alcohol-based hand rub be inadequate?
++
++
++
A. A 20-year-old renal transplant recipient with varicella pneumonia
++
++
B. A 40-year-old man with MRSA furunculitis
++
++
C. A 35-year-old woman with advanced HIV and cavitary pulmonary tuberculosis
++
++
D. A 54-year-old quadriplegic man admitted with a urinary tract infection due to extended-spectrum beta lactamase producing bacteria
++
++
E. A 78-year-old nursing home resident with recent antibiotic use and Clostridium difficile infection
+
++
++
(Chap. 17) Nosocomial infections have reservoirs and sources just as do community-acquired pathogens. In hospitalized patients, cross-contamination (i.e., indirect spread of organisms from one patient to the next) accounts for many nosocomial infections. While hand hygiene is uniformly recommended for health care practitioners, adherence to hand washing is low often due to time pressure, inconvenience, and skin damage. Because of improved adherence, alcohol-based hand rubs are now recommended for all heath care workers except when hands are visibly soiled or after care of a patient with C. difficile infection, whose spores may not be killed by alcohol and thus require thorough hand wash with soap and water.
++
++
++
A 32-year-old woman is admitted to the hospital complaining of right thigh pain. She is treated empirically with oxacillin intravenously for a cellulitis. The admitting physician notes that the degree of pain appears to be disproportionate to the amount of overlying cellulitis. Over the course of the next 24 hours, the patient develops profound septic shock complicated by hypotension, acute renal failure, and evidence of disseminated intravascular coagulation. A CT scan of her right leg demonstrates a collection of fluid with gas in the deep fascia of her right leg. Emergent surgical evacuation is planned. What changes to the patient’s antibiotic therapy would be recommended?
++
++
++
A. Continue oxacillin and add clindamycin
++
++
B. Continue oxacillin and add clindamycin and gentamicin
++
++
C. Discontinue oxacillin and add clindamycin, vancomycin, and gentamicin
++
++
D. Discontinue oxacillin and add piperacillin/tazobactam and vancomycin
++
++
E. Discontinue oxacillin and add vancomycin and gentamicin
+
++
++
(Chap. 18) Necrotizing fasciitis is a life-threatening infection that leads to extensive necrosis of the subcutaneous tissue and fascia. It is most commonly caused by group A streptococci and mixed a mixed facultative and anaerobic flora. Recently, there have been an increasing number of cases of necrotizing fasciitis due to community acquired methicillin-resistant Staphylococcus aureus. Risk factors include diabetes mellitus, intravenous drug use, and peripheral vascular disease. The infection often arises at a site of minimal trauma, and the physical findings initially are minimal in comparison to the severity of pain and fever. The mortality rate for necrotizing fasciitis is between 15–34%, but rise to as high as 70% if toxic shock syndrome is present. Wide surgical debridement of the affected tissue is necessary, and without surgery, the mortality is near 100%. A high index of clinical suspicion is important for selecting the appropriate antibiotic therapy and early consultation of surgery. The initial antibiotics should cover the typical organisms and include vancomycin 15 mg/kg IV every 12 hours, clindamycin 600 mg IV every 8 hours, and gentamicin 5 mg/kg every 8 hours intravenously.
++
++
++
All of the following statements regarding the epidemiology of sepsis and septic shock are true EXCEPT:
++
++
++
A. Gram-positive bacteria are the most commonly isolated causative organisms in sepsis syndromes
++
++
B. In individuals with septic shock, blood cultures are positive in 40–70% of cases
++
++
C. Most cases of sepsis occur in individuals with significant underlying illness
++
++
D. Respiratory infections are the most common cause of sepsis syndromes
++
++
E. The annual incidence of severe sepsis has increased over the past 30 years with a current incidence of ~3 per 1000 population
+
++
++
(Chap. 19) Sepsis and septic shock represent a harmful host response to infection and is a contributing factor to more than 200,000 deaths in the United States each year. Over the past 30 years, the incidence of sepsis has risen along with our aging population. Annually, there are more than 750,000 cases of sepsis yearly with an incidence of 3 per 1000 population. Most cases of sepsis occur in individuals with underlying illness. The incidence also increases with age and increasing comorbidities. Other risk factors for sepsis include immunocompromised state and indwelling vascular or mechanical devices. The most common source of infection in individuals with sepsis is the lungs, accounting for 64% of cases of sepsis. Microbial invasion of the bloodstream is not required for an individual to develop the systemic response that leads to multi-organ dysfunction in sepsis. Blood cultures are only positive in 40–70% of cases of septic shock and ~20–40% of cases of severe sepsis. When a culture is positive from any site, the most common organisms are Gram-negative bacteria (62%) with Pseudomonas aeruginosa and Escherichia coli being the most common. Gram-positive bacteria are present in approximately 47% of cases, and 19% occurred in people infected with fungi. As the numbers add up to >100%, it should be noted that multiple bacteria may be implicated in sepsis.
++
++
++
A 32-year-old woman presents to her primary care physician complaining of nasal congestion and drainage and headache. Her symptoms originally began about 7 days ago with rhinorrhea and sore throat. For the past 5 days, she has been having increasing feelings of fullness and pressure in the maxillary area that is causing her headaches. The pressure is worse when she bends over and she also notices it while lying in bed at night. She is otherwise healthy and has not had fevers. On physical examination, there is purulent nasal drainage and pain with palpation over bilateral maxillary sinuses. What is the best approach to ongoing management of this patient?
++
++
++
A. Initiate therapy with amoxicillin 500 mg three times daily for 10 days
++
++
B. Initiate therapy with levofloxacin 500 mg daily for 10 days
++
++
C. Perform a sinus aspirate for culture and sensitivities
++
++
++
++
E. Treat with oral decongestants and nasal saline lavage
+
++
++
(Chap. 20) Acute sinusitis is a common complication of upper respiratory tract infections and is defined as sinusitis lasting <4 weeks duration. Acute sinusitis typically presents nasal drainage and congestion, facial pain or pressure, and headache that are worse with lying down or bending forward. Presence of purulent drainage does not differentiate bacterial from viral causes of sinusitis. The vast majority of cases of acute sinusitis are due to viral infection. However, when patients with acute sinusitis present to a medical profession, antibiotics are prescribed more than 85% of the time. Indeed, this should not be the preferred treatment as most cases improve without antibiotic therapy. Rather the initial approach to a patient with acute sinusitis should be symptomatic treatment with nasal decongestants and nasal saline lavage. If a patient has a history of allergic rhinitis or chronic sinusitis, nasal glucocorticoids can be prescribed as well. Antibiotic therapy is recommended in adults for symptom duration greater than 7–10 days and in children greater than 10–14 days. In addition, any patient with concerning features such as unilateral or focal facial pain or swelling should be treated with antibiotics. The initial antibiotic of choice for acute sinusitis is amoxicillin 500 mg orally three times daily or 875 mg twice daily. If a patient has prior exposure to antibiotics within the past 30 days or treatment failure, a respiratory fluoroquinolone can be given. Ten percent of individuals do not respond to initial antibiotic therapy. In this case, one can consider referral to otolaryngology for sinus aspiration and culture. Radiologic imaging of the sinuses is not recommended for evaluation of acute disease unless the sinusitis is nosocomially acquired as the procedures (CT or radiograph) do not differentiate between bacterial or viral causes.
++
++
++
Chronic dialysis is associated with an increased risk of health care-associated pneumonia caused by which of the following?
++
++
++
++
++
++
++
C. Methicillin-resistant staphylococcus aureus (MRSA)
++
++
D. Multidrug-resistant enterobacteriaceae
++
++
E. Pseudomonas aeruginosa
+
++
++
(Chap. 21) There are multiple clinical conditions associated with health care-associated pneumonia (HCAP). Residence in a nursing home or extended-care facility, hospitalizations for two or more days in the prior 3 months, and current hospitalizations that have been 48 hours or longer are associated with HCAP due to MRSA, Pseudomonas aeruginosa, Acinetobacter species, and multidrug-resistant enterobacteriaceae. HCAP due to Pseudomonas aeruginosa is also associated with antibiotic therapy in the preceding 3 months. Chronic dialysis, home infusion therapy, and home wound care place patients at risk for HCAP due to MRSA. Having a family member with a multidrug-resistant infection is associated with HCAP due to both MRSA and multidrug-resistant enterobacteriaceae.
++
++
++
All of the following statements regarding lung abscesses are true EXCEPT:
++
++
++
A. Lemierre’s syndrome is a lung abscess due to septic thrombophlebitis originating in the pharynx
++
++
B. Lung abscesses are typically characterized by a >2 cm single dominant cavity
++
++
C. Primary lung abscesses are often principally caused by anaerobic bacteria
++
++
D. Primary lung abscesses typically are related to oropharyngeal aspiration
++
++
E. Radiographically primary lung abscesses most commonly involve the middle lobe and the lingual
+
++
++
(Chap. 22) Lung abscess represents necrosis and cavitation of the lung following microbial infection. Lung abscesses can be single or multiple but usually are marked by a single dominant cavity >2 cm in diameter. Although the incidence of lung abscesses has decreased in the postantibiotic era, they are still a source of significant morbidity and mortality. Lung abscesses are usually characterized as either primary (~80% of cases) or secondary. Primary lung abscesses usually arise from aspiration, are often caused principally by anaerobic bacteria, and occur in the absence of an underlying pulmonary or systemic condition. Patients at particular risk for aspiration, such as those with altered mental status, alcoholism, drug overdose, seizures, bulbar dysfunction, prior cerebrovascular or cardiovascular events, or neuromuscular disease, are most commonly affected. In addition, patients with esophageal dysmotility or esophageal lesions (strictures or tumors) and those with gastric distention and/or gastroesophageal reflux, especially those who spend substantial time in the recumbent position, are at risk for aspiration. It is widely thought that colonization of the gingival crevices by anaerobic bacteria or microaerophilic streptococci (especially in patients with gingivitis and periodontal disease), combined with a risk of aspiration, is important in the development of lung abscesses. Secondary lung abscesses arise in the setting of an underlying condition, such as a postobstructive process (e.g., a bronchial foreign body or tumor) or a systemic process (e.g., HIV infection or another immunocompromising condition). Lemierre’s syndrome, in which an infection begins in the pharynx (classically involving Fusobacterium necrophorum) and then spreads to the neck and the carotid sheath (which contains the jugular vein) to cause septic thrombophlebitis. Because most cases of lung abscess occur in the context of reclining aspiration, the dependent lobes of the lung are most vulnerable. The dependent lobes include the superior segment of the lower lobes and the posterior segment of the upper lobes with the right lung more commonly involved than the left lung. The middle lobe and the lingula are the most ventral lobes and are therefore non-dependent in the supine position.
++
++
++
A 51-year-old woman presents complaining of a daily cough productive of thick green sputum. The cough is worse when she first wakes in the morning. At this time, there are occasionally streaks of blood in the sputum. Her cough began about 7 years ago and has been progressively worse with production of increasing volume of sputum. She currently estimates that she brings up about ½ cup of sputum daily. She reports frequently requiring antibiotics for both lower respiratory tract infections as well as sinus infections. Bilateral coarse crackles are heard in the lower lung zones. No clubbing is present. Pulmonary function tests demonstrate an FEV1 1.68 L (53.3% predicted), FVC 3.00 L (75% predicted) and FEV1/FVC ratio of 56%. A sputum culture grows Pseudomonas aeruginosa. What test would you perform next in the evaluation of this patient?
++
++
++
++
++
++
++
++
++
D. High resolution chest tomography
++
++
E. Sweat chloride testing
+
++
++
(Chap. 23) Bronchiectasis is a common disorder that refers to an irreversible dilation of the airways that affect the lung in a focal or diffuse pattern. Historically, bronchiectasis has been characterized pathologically as cylindrical, varicose, or cystic in nature. There are numerous causes of bronchiectasis including infectious, inherited, immunologic, and idiopathic causes. The most common cause worldwide of bronchiectasis is post-infectious following tuberculosis infection. However, in developed countries, other causes are more common. The most common cause in developed countries is cystic fibrosis (CF), which develops clinically significant bronchiectasis in late adolescence or early adulthood. Most children with CF are diagnosed currently through newborn screening programs which were introduced in all states within the past decade. However, adults may continue to present with milder forms of the disease so clinicians should continue to have a high degree of clinical suspicion for CF when an individual presents with a new diagnosis of bronchiectasis. Epidemiologically, individuals presenting with CF bronchiectasis will typically be younger than individuals with non-CF bronchiectasis. In contrast, non-CF bronchiectasis more commonly affects nonsmoking women older than 50 years. The clinical presentation of bronchiectasis is a daily cough productive of thick tenacious sputum. Physical examination demonstrates both crackles and wheezing on examination. In CF, the disease findings are more predominant in the upper lobes whereas in certain other causes (chronic aspiration, immunoglobulin deficiency), there is a lower lobe predominance. Clubbing is variably present and generally only seen in more advance disease. The diagnosis of bronchiectasis is made by determined the presence of the disease on a chest CT imaging. Chest radiography is not sensitive for the diagnosis of bronchiectasis particularly early in the disease process. It may show “tram tracks” indicating dilated airway. High resolution chest CT is the diagnostic modality of choice for confirming diagnosis. Findings included demonstration of dilated, non-tapering airways that may be filled with mucus. In addition, signet-ring signs may also be seen where the airway is greater than 1.5 times the size of the adjacent blood vessel. Other findings include bronchial wall thickening, inspissated secretion with a “tree-in-bud” patter, and cysts emanating from the bronchial wall. Once bronchiectasis has been confirmed on CT imaging, other tests may be indicated in this patient to determine the etiology of the bronchiectasis. Testing should be guided by the history and physical examination and may include sputum culture for mycobacteria and fungal organisms, immunoglobulin levels, autoimmune panel, sweat chloride testing, nasal nitric oxide testing (for primary ciliary dyskinesia), bronchoscopy, and/or tests of swallow function. Despite thorough work up, in as many as 25–50% of individuals referred for work up of bronchiectasis, no specific cause is ever identified.
++
++
++
Which of the following is the most common cause of native valve infective endocarditis in the community?
++
++
++
A. Coagulase-negative staphylococci
++
++
B. Coagulase-positive staphylococci
++
++
++
++
D. Fastidious Gram-negative coccobacilli
++
++
E. Non-enterococcal streptococci
+
++
++
(Chap. 24) The etiologic agents of infective endocarditis vary by host (see Table 24-1). Community acquired native valve endocarditis remains an important clinical problem, particularly in the elderly. In those patients, streptococci (viridans spp., S. gallolyticus, other non-group A and other group streptococci, and abiotrophia spp.) account for approximately 40% of cases. Staphylococcus aureus (28%) is next most common. Enterococci, HACEK group, coagulase-negative, and culture-negative cases each account for <10% of community acquired native valve cases. In health care-associated, injection drug use associated, and >12 month old prosthetic valve endocarditis, staphylococcus aureus is most common. Coagulase-negative staphylococcus is the most common organism in prosthetic valve endocarditis <12 months. Enterococci cause endocarditis in approximately 10–15% of cases in health care-associated, 2–12 month prosthetic valve, and injection drug use cases. Culture negative endocarditis accounts for 5–10% of cases in the aforementioned clinical scenarios.
++
++
++
++
All of the following ECG characteristics will aid in differentiating acute pericarditis from acute myocardial infarction EXCEPT:
++
++
++
A. Absence of the development of Q-waves
++
++
B. Concave-shape to ST-elevations
++
++
++
++
++
++
E. T-wave inversions after return of the ST segments to baseline
+
++
++
(Chap. 25) The electrocardiogram (ECG) in acute pericarditis without massive effusion usually displays changes secondary to acute subepicardial inflammation. It typically evolves through four stages. In stage 1, there is widespread elevation of the ST segments, often with upward concavity, involving two or three standard limb leads and V2–V6, with reciprocal depressions only in aVR and sometimes V1. Also, there is depression of the PR segment below the TP segment, reflecting atrial involvement. Usually there are no significant changes in QRS complexes. After several days, the ST segments return to normal (stage 2), and only then, or even later, do the T waves become inverted (stage 3). Weeks or months after the onset of acute pericarditis, the ECG returns to normal (stage 4). In contrast, in AMI, ST elevations are convex, and reciprocal depression is usually more prominent; these changes may return to normal within a day or two. Q waves may develop, with loss of R-wave amplitude, and T-wave inversions are usually seen within hours before the ST segments have become isoelectric. ST-elevation in V2 may be seen in either acute pericarditis or myocardial infarction.
++
++
++
All of the following infectious clinical syndromes typically cause vesicular lesions EXCEPT:
++
++
++
++
++
B. Hand-foot-and-mouth disease
++
++
++
++
++
++
+
++
++
(Chap. 26) Vesicle formation due to infection may be caused by viral proliferation within the epidermis. In varicella and variola, viremia precedes the onset of a diffuse centripetal rash that progresses from macules to vesicles, then to pustules, and finally to scabs over the course of 1–2 weeks. Vesicles of varicella have a “dewdrop” appearance and develop in crops randomly about the trunk, extremities, and face over 3–4 days. Herpes zoster occurs in a single dermatome; the appearance of vesicles is preceded by pain for several days. Zoster (shingles) may occur in persons of any age but is most common among immunosuppressed individuals and elderly patients, whereas most cases of varicella occur in young children. Cold sores are typically caused by primary infection or reactivation of herpes simplex virus (HSV). Vesicles due to HSV are found on or around the lips (HSV-1) or genitals (HSV-2) but also may appear on the head and neck of young wrestlers (herpes gladiatorum) or on the digits of health care workers (herpetic whitlow). Recurrent herpes labialis (HSV-1) and herpes genitalis commonly follow primary infection. Hand-foot-and-mouth disease is caused by coxsackievirus A16 and characteristically causes vesicles on the hands, feet, and mouth of children. Rickettsialpox begins after mite-bite inoculation of Rickettsia akari into the skin. A papule with a central vesicle evolves to form a 1- to 2.5-cm painless crusted black eschar with an erythematous halo and proximal adenopathy. While more common in the northeastern United States and the Ukraine in 1940–1950, rickettsialpox has recently been described in Ohio, Arizona, and Utah. Scalded skin syndrome, typically caused by a staphylococcus aureus toxin, presents with bullae, not vesicles. (See Figure 26-1.)
+
++
++
Figure 26-1: Structural components of the skin and soft tissue, superficial infections, and infections of the deeper structures. The rich capillary network beneath the dermal papillae plays a key role in the localization of infection and in the development of the acute inflammatory reaction.
++
++
++
A 45-year-old man with a history of alcoholism and presumed cirrhosis is brought to the emergency room by his friend complaining of 2–3 days of increasing lethargy and confusion. He has not consumed alcohol in the past 2 years. He currently takes no medications and works at home as a video game designer. He has no risk factors for HIV. He was referred by his primary care physician for a liver transplant evaluation and is scheduled to begin his evaluation next month. His vital signs included blood pressure 90/60, heart rate 105 beats/min, temperature 38.5°C, respiratory rate 10/min with O2 saturation 97% on room air. He is somnolent but is able to answer questions accurately. His skin is notable for many spider telangiectasias and palmar erythema. He has a distended diffusely tender abdomen with a positive fluid wave. Paracentesis reveals slightly cloudy fluid with WBC 1000/ul and 40% neutrophils. His blood pressure increases to 100/65 and his heart rate decreases to 95 beats/min after 1 L intravenous fluids. Which of the following statements regarding his condition and treatment is true?
++
++
++
A. Fever is present in >50% of cases
++
++
B. Initial empiric therapy should include metronidazole or clindamycin for anaerobes
++
++
C. The diagnosis of primary (spontaneous) bacterial peritonitis is not confirmed because the percentage of neutrophils in the peritoneal fluid is <50%
++
++
D. The mostly causative organism for his condition is enterococcus
++
++
E. The yield of peritoneal fluid cultures for diagnosis is >90%
+
++
++
(Chap. 29) Primary (spontaneous) bacterial peritonitis (PBP) occurs when the peritoneal cavity becomes infected without an apparent source of contamination. PBP occurs most often in patients with cirrhosis, usually with preexisting ascites. The bacterial likely invade the peritoneal fluid because of poor hepatic filtration in cirrhosis. While fever is present in up to 80% of cases, abdominal pain, acute onset, and peritoneal signs are often absent. Patients may present with nonspecific findings such as malaise or worsening encephalopathy. A neutrophil count in peritoneal fluid of >250/ul is diagnostic; there is no % neutrophil differential threshold. Diagnosis is often difficult as peritoneal cultures are often negative. Blood cultures may reveal the causative organism. The most common organisms are enteric Gram-negative bacilli, but Gram-positive cocci are often found. Anaerobes are not common (in contrast to secondary bacterial peritonitis) and empiric antibiotics targeting them are not necessary if PBP is suspected. Third generation cephalosporins or piperacillin/tazobactam are reasonable initial empiric therapy. Diagnosis requires exclusion of a primary intraabdominal source of peritonitis.
++
++
++
A 78-year-old woman presents to the hospital from her nursing home with complaints of diarrhea. She has been in the nursing home for the past 5 years following a stroke with residual right sided hemiplegia. She was recently treated with ceftriaxone for pyelonephritis due to E. coli. Yesterday, she developed a temperature of 100.6°F with a complaint of diffuse abdominal pain. Over the past 24 hours, she has had worsening abdominal pain and abdominal distention. In addition, she has had eight bowel movements. The bowel movements are loose and have become bloody. Six months ago she was treated with oral metronidazole for a documented C. difficile infection. Additionally, her past medical history is significant for cerebrovascular disease, atrial fibrillation, coronary artery disease requiring angioplasty, hypertension, and hyperlipidemia. On presentation to the hospital, she appears uncomfortable and has a temperature of 101.2°F. Her blood pressure is 98/60 and heart rate is 115 beats/min. Her abdomen appears distended and tympanitic with diffuse tenderness to palpation. An abdominal x-ray shows distention of the colon with ileus. Initial laboratory examination shows a white blood cell count of 27,200/mm3 with 92% neutrophils and 3% band forms. Her hemoglobin is 9.2 g/dL and hematocrit is 28.1%. One month ago her hemoglobin was 10.1 g/dL. Given her recent antibiotic use, you consider the possibility of C. difficile infection. Which of the following findings is unlikely to be found in C. difficile infection?
++
++
++
++
++
++
++
++
++
++
++
E. Recurrence after therapy
+
++
++
(Chap. 31) Clostridium difficile infection is a common gastrointestinal illness that is most commonly associated with antimicrobial use and subsequent disruption of normal colonic flora. Cases of C. difficile infection have been rising since the early 2000s, tripling between the years of 2000 to 2005. Epidemiologically, fecal colonization with C. difficile is ≥20% after 1 week of hospitalization, but remains lows at 1–3% among community-dwelling adults. The most common presentation of C. difficile infection is increased stooling with stools that may range from soft and unformed to profuse watery diarrhea. Stools are almost never grossly bloody. An individual infected with C. difficile may have as many as 20 bowel movements per day. Clinically, fever is present in up to 28% of cases, abdominal pain in 22% and leukocytosis in 50%. An ileus is often seen on abdominal radiograph. The ileus may be adynamic in origin in 20% of cases. An adynamic ileus occurs when there is an ileus on radiograph with cessation of stooling. C. difficile infection recurs after treatment in 15–30% of patients. In this patient, the bloody stools, fever, and abdominal pain suggests ischemic colitis, and the patient has several risk factors for this including atrial fibrillation and cardiovascular disease.
++
++
++
All of the following are potential causes of appendix obstruction and appendicitis EXCEPT:
++
++
++
++
++
++
++
++
++
++
++
+
++
++
(Chap. 32) Obstruction of the appendiceal lumen is believed to typically result in appendicitis. Although obstruction is most commonly caused by fecalith, which results from accumulation and insipissation of fecal matter around vegetable fibers, other causes have been described. These other potential causes include enlarged lymphoid follicles associated with viral infection (e.g., measles), insipissated barium, worms (e.g., pinworms, Ascaris, and Taenia), and tumors such as carcinoma or carcinoid. Cholelithiasis is a common cause of acute pancreatitis.
++
++
++
Which of the following statements regarding the epidemiology of and risk factors for urinary tract infections (UTI) is true?
++
++
++
A. About 1/3 of all women will experience at least one UTI in their lifetime
++
++
B. Across all ages, UTI is 2–3 times more common among females
++
++
C. Asymptomatic bacteriuria is a common and incidental finding in pregnancy that does not require treatment
++
++
D. Contrary to popular wisdom, sexual intercourse is not a risk factor for UTI
++
++
E. In infancy, UTI is more common among males than females
+
++
++
(Chap. 33) Urinary tract infection (UTI) is one of the most common infections seen in primary care. The term UTI encompasses a number of clinical entities including asymptomatic bacteriuria, cystitis, pyelonephritis, and prostatitis. Acute cystitis is the most common form of UTI that is diagnosed. Except in infancy and in the elderly, UTI is far more common in women. Due to the greater incidence of congenital urinary tract anomalies in the males, the incidence of UTI is greater in males in infancy. Between the ages of 1 and 50 years, women far outnumber men with UTI. After the age of 50 years, men have increasing incidence of UTI, and overall the incidence in men nears that of women. Over a lifetime, 50–80% of women will experience at least one UTI. Risk factors for UTI in women include prior history of UTI, diabetes mellitus, incontinence, sexual activity, and use of a diaphragm with spermicide. About 20–30% of women who experience a UTI will have recurrent episodes of UTI. Recent sexual activity is also temporally related to UTI. One episode of intercourse in the preceding week is associated with a relative risk of UTI of 1.4. With five episodes of intercourse in a week, the relative risk increases to 4.8. Men who experience UTI typically have a structural abnormality that contributes to the development of infection. The most common abnormality in men is prostatic hypertrophy that causes urinary obstruction. Lack of circumcision also increases risk of UTI in men as the E. coli may colonize the glans and prepuce of an uncircumcised male. In pregnancy, women should be screened and treated for asymptomatic bacteriuria. The presence of asymptomatic bacteria in a pregnant woman is associated with development of pyelonephritis, preterm birth, and perinatal fetal death.
++
++
++
All of the following are common causes of urethritis in men EXCEPT:
++
++
++
++
++
++
++
++
++
D. Trichomonis vaginialis
++
++
E. Ureaplasma urealyticum
+
++
++
(Chap. 35) Common causes of urethral discomfort and discharge in men include Chlamydia trachomatis, Neisseria gonorrhoeae, Mycoplasma genitalium, Ureaplasma urealyticum, Trichomonas vaginalis, and herpes simplex virus. Gardnerella is the usual cause of bacterial vaginosis in women and is not a pathogen in men.
++
++
++
All of the following are predisposing conditions that increase the risk for development of acute bacterial meningitis due to Streptococcus pneumoniae EXCEPT:
++
++
++
++
++
B. Pneumococcal pneumonia
++
++
C. Pneumococcal sinusitis
++
++
++
++
+
++
++
(Chaps. 36 and 44) S. pneumoniae is the most common cause of meningitis in adults >20 years of age, accounting for nearly half the reported cases (1.1 per 100,000 persons per year). There are a number of predisposing conditions that increase the risk of pneumococcal meningitis, the most important of which is pneumococcal pneumonia. Additional risk factors include coexisting acute or chronic pneumococcal sinusitis or otitis media, alcoholism, diabetes, splenectomy, hypogammaglobulinemia, complement deficiency, and head trauma with basilar skull fracture and CSF rhinorrhea. The mortality rate remains ~20% despite antibiotic therapy. Pregnancy, age >60 years, and immunocompromised status are important risk factors for meningitis due to Listeria monocytogenes.
++
++
++
A 45-year-old woman presents with an 8 week history of new onset headache that is persistent and daily. She describes the pain as a diffuse ache and rates it 6–7 out of 10. It has been worsening over time. She had seen her primary doctor who reassured her that her physical examination was normal and he prescribed ibuprofen 600 mg prn pain. This treatment did not alleviate her symptoms. Yesterday, she awoke double vision and a facial droop. The patient lives in Pennsylvania and hikes frequently along the Appalachian Trial. She does not recall any tick bites. She has intermittently had joint pain over this past month. She denies rash. She presents to the emergency room for further evaluation. On physical examination, the patient has a temperature of 99.4°F. Vital signs are normal. Neurologic examination demonstrates a complete right facial droop. The left eye fails to abduct when looking laterally although the right eye has full range of motion. Which of the following findings is most likely to be present in the cerebrospinal fluid?
++
++
++
A. Elevated angiotensin converting enzyme level
++
++
B. Elevated neutrophil count
++
++
C. Elevated protein level
++
++
++
++
E. Positive venereal disease research laboratory (VDRL) test
+
++
++
(Chap. 37) Chronic meningitis has many causes, including a variety of bacteria, mycobacteria, fungi, viruses, and parasites. In addition, many non-infectious etiologies can cause chronic meningitis as well, and these include systemic lupus erythematosus, sarcoidosis, Behçet’s syndrome, Mollaret’s meningitis, drug hypersensitivity, and malignancy among others. Chronic meningitis is diagnosed when there is a characteristic syndrome of meningitis that has produced symptoms for ≥4 weeks and is associated with inflammation in the cerebrospinal fluid (CSF). Clinically, a patient with chronic meningitis typically presents with headache, neck pain, or back pain. In this patient also presenting with cranial nerve VI and VII palsy, the lesion can be localized to a basal meningitis affecting cranial nerve roots. Given the patient’s geographic location and avidity for hiking, she is at risk for infection with Lyme disease caused by the organism Borrelia burgdorferi. Lyme disease causing meningitis typically occurs weeks or months after the tick bite, and many patients cannot recall having a tick bite. Likewise, patients may not recall having the typical target shaped erythema migrans rash which occurs early in the disease process. Other features of early disseminated Lyme disease include arthralgias/arthritis, carditis, conjunctivitis or iritis, radiculopathy, and lymphadenopathy. Diagnosis of Lyme disease as a cause of chronic meningitis relies primarily on demonstrating a positive serum Lyme titer and Western blot. The CSF findings are non-specific including increased mononuclear cells and elevated protein typically to not greater than 200–300 mg/dL. CSF glucose would be normal. Demonstration of a positive antibody to B. burgdorferi in the CSF is highly specific for disease, but may be negative. Other diseases that can present with a basal meningitis picture include neurologic sarcoidosis, malignancy, and granulomatosis with polyangiitis. An elevated CSF angiotensin converting enzyme level may be seen in sarcoidosis, but is neither sensitive nor specific for diagnosing disease. Treponema pallidum is a spirochete like B. burgoderi and is the causative organism for syphilis. Untreated syphilis can result in chronic meningitis as well with symptoms of dementia rather than cranial nerve palsies. Because the causative organisms for both Lyme disease and syphilis are spirochetes, there is a degree of cross-reactivity on serum Lyme titers, but a positive CSF venereal disease research (VDRL) test is only positive in syphilis.
++
++
++
A 33-year-old woman seeks an additional medical opinion after seeing multiple physicians in the past 3 years. She describes unrelenting fatigue that has lasted for approximately 2 years to the point where she no longer exercises and is in danger of losing her job as a copy editor. Her sleep is typically unsettled and no matter how much she sleeps, she reports never feeling refreshed. She dates the onset of the fatigue to an episode on serologically confirmed mononucleosis 3 years prior. Her husband agrees that “she has never recovered from that episode”. She has tried antidepressants and various supplements with no benefit. Her past medical history is unremarkable other than having anorexia and depression as a teenager. She says she is fully recovered from that since college. Her physical examination is unremarkable other than a resting heart rate of 95 beats/min. She has a normal BMI. Which of the following is an exclusion to the diagnosis of chronic fatigue syndrome?
++
++
++
A. History of anorexia and depression
++
++
B. History of mononucleosis
++
++
++
++
D. Resting heart fate >90 beats/min
++
++
+
++
++
(Chap. 38) Chronic fatigue syndrome (CFS) is a disorder characterized by persistent and unexplained fatigue resulting in severe impairment in daily functioning. Besides intense fatigue, most patients with CFS report concomitant symptoms such as pain, cognitive dysfunction, and unrefreshing sleep. Additional symptoms can include headache, sore throat, tender lymph nodes, muscle aches, joint aches, feverishness, difficulty sleeping, psychiatric problems, allergies, and abdominal cramps. Criteria for the diagnosis of CFS have been developed by the U.S. Centers for Disease Control and Prevention (see Table 38-1). CFS is seen worldwide, with adult prevalence rates varying between 0.2% and 0.4%. In the United States, the prevalence is higher among women (~75% of cases), members of minority groups (African and Native Americans), and individuals with lower levels of education and occupational status. The mean age of onset is between 29 and 35 years. Many patients probably go undiagnosed and/or do not seek help. There are numerous hypotheses about the etiology of CFS; there is no definitively identified cause. Physical inactivity and trauma in childhood tend to increase the risk of CFS in adults. Neuroendocrine dysfunction may be associated with childhood trauma, reflecting a biological correlate of vulnerability. Psychiatric illness and physical hyperactivity in adulthood raise the risk of CFS in later life. Twin studies suggest a familial predisposition to CFS, but no causative genes have been identified. Physical or psychological stress may elicit the onset of CFS. Most patients report an infection (usually a flulike illness or infectious mononucleosis) as the trigger of their fatigue. Relatively high percentages of CFS cases follow Q fever and Lyme disease.
++
++
++
++
A 22-year-old man with cystic fibrosis with admitted to the hospital for an exacerbation of his underlying disease. He is known to be colonized with Pseudomonas aeruginosa. His empiric antibiotic regimen is ceftazidime 2 g IV every 8 hours and tobramycin dosed 10 mg/kg once daily. Which statement best describes the pharmacokinetics and pharmacodynamics of this combination of agents in the treatment of a patient with cystic fibrosis?
++
++
++
A. Aminoglycoside antibiotics such as tobramycin kill bacteria in a time-dependent fashion and thus need higher concentrations of antibiotics
++
++
B. Assessing trough levels of aminoglycosides will help determine if the appropriate drug level to achieve optimal killing of organisms has occurred
++
++
C. For beta-lactam antibiotics, the duration of time with a concentration of drug above the minimal inhibitory concentration (MIC) determines the killing effect
++
++
D. Lower doses of antibiotics is typically required because cystic fibrosis patients have a higher volume of distribution of antibiotics
++
++
E. Steady state concentration of the drugs will be reached in three to four half-lives of the drug
+
++
++
(Chap. 41) Pharmacokinetics and pharmacodynamics both play important roles in understanding the appropriate use of antibiotics. Pharmacokinetics refers to the disposition of a drug in the body and includes the absorption, distribution, metabolism, and excretion of the drug. Intravenous use of a drug leads to 100% of absorption of a drug. Other routes of drug administration yield varying degrees of bioavailability and are subject to first-pass effects. Once a drug is in the body, distribution refers to how the drug is passed between the circulation and tissues, and it is important to consider the volume of distribution of a drug. This refers to the amount of drug in the body at a given time relative to the measured serum concentration. Patients with cystic fibrosis are known to have a higher volume of distribution of medications with more rapid drug clearance. Thus, higher doses of antibiotics are often required. Some medications require monitoring to determine if appropriate drugs levels are attained to achieve effective killing but while minimizing side effects. It is important to consider drug metabolism and excretion when determining the appropriate time to assess drug levels. Generally, five to seven half-lives of the drug are required for levels to achieve steady state when multiple doses of a medication are given in a time frame short than the half-life itself. Trough levels prior to drug administration are performed to ensure that the drug is not accumulating and to prevent side effects. Pharmacodynamics describes the relationship between the serum concentration that achieves desired drug effect and serum concentration that can produce toxic effects of the drug. When considering the pharmacodynamics of an antibiotic, it is important to consider whether the drug achieves efficacy through time- or concentration-dependent killing. Beta-lactams are an example of a drug that kills in a time-dependent fashion. Thus, these agents do not require high peak levels for efficacy rather the important factor in achieving effect is to determine the length of time the drug will be maintained at concentrations greater than the minimal inhibitory concentration (MIC). The longer the concentration of a beta-lactam remains above the MIC during the dosing interval, the greater the killing effect, and using prolonged infusions of beta-lactams has been utilized as a way to improve time-dependent killing. In contrast, concentration-dependent killing requires higher peak concentration of drug to achieve the desired effect. Aminoglycosides are antibiotics that use concentration dependent pharmacodynamics.
++
++
++
All of the following statements regarding the pneumococcus (S. pneumoniae) are true EXCEPT:
++
++
++
A. Asymptomatic colonization does not occur
++
++
B. Infants (<2 years old) and elderly are at greatest risk of invasive disease
++
++
C. Pneumococcal vaccination has impacted the epidemiology of disease
++
++
D. The likelihood of death within 24 hours of hospitalization for patients with invasive pneumococcal pneumonia has not changed since the introduction of antibiotics
++
++
E. There is a clear association between prior viral upper respiratory infection and secondary pneumococcal pneumonia
+
++
++
(Chap. 42) Pneumococcal infections, particularly pneumonia, remain a worldwide public health problem. Intermittent colonization of the nasopharynx by pneumococcus transmitted by respiratory droplet is common and is the likely reservoir for invasive disease. Infants and elderly are at greatest risk of developing invasive pneumococcal disease (IPD) and death. In the developed world, children are the most common source of pneumococcal transmission. By 1 year old, 50% of children have had at least one episode of colonization. Prevalence studies show carriage rates of 20–50% in children up to 5 year old and up to 15% for adults. These numbers approach 90% for children and 40% for adults in the developing world. Pneumococcal vaccination has dramatically impacted the epidemiology with reduced IPD in the United States attributable to reductions in serotypes included in the vaccine. Similar reductions have been observed in other countries implementing routine childhood vaccinations; however, in certain populations (Alaska native populations and the UK) the reduction in vaccine covered serotype cases has been offset by increases in nonvaccine serotypes. Case fatality rates due to pneumococcal pneumonia vary by host factors, age, and access to care. Interestingly, there appears to be no reduction in case fatality during the first 24 hours of hospitalization since the introduction of antibiotics. This is likely due to the development of severe multiorgan failure as a result of severe infection. Appropriate care in an intensive setting can reduce case fatality rate for severe infection. Outbreaks of disease are well recognized in crowded settings with susceptible individuals, such as infant day-care facilities, military barracks, and nursing homes. Furthermore, there is a clear association between preceding viral respiratory disease (especially but not exclusively influenza) and risk of secondary pneumococcal infections. The significant role of pneumococcal pneumonia in the morbidity and mortality associated with seasonal and pandemic influenza is increasingly recognized.
++
++
++
Which of the following statements regarding Staphylococcus aureus is true?
++
++
++
A. Infections due to S. aureus among colonized individuals is almost always due to a strain distinct from their colonizing strain
++
++
B. S. aureus is always a pathogen when detected in any human culture
++
++
C. The anterior nares and oropharynx are the most common sites of colonization with S. aureus
++
++
D. The colonization rate in the population is independent of patient factors
++
++
E. The majority of people in the United States are colonized with S. aureus
+
++
++
(Chap. 43) S. aureus is both a commensal and an opportunistic pathogen. Approximately 30% of healthy persons are episodically colonized with S. aureus, with a smaller percentage (~10%) persistently colonized. The rate of colonization is elevated among insulin-dependent diabetics, HIV-infected patients, patients undergoing hemodialysis, injection drug users, and individuals with skin damage. The anterior nares and oropharynx are frequent sites of human colonization, although the skin (especially when damaged), vagina, axilla, and perineum may also be colonized. These colonization sites serve as a reservoir for future infections. Transmission of S. aureus most frequently results from direct personal contact. Colonization of different body sites allows transfer from one person to another during contact. Spread of staphylococci in aerosols of respiratory or nasal secretions from heavily colonized individuals has also been reported. Most individuals who develop S. aureus infections become infected with a strain that is already a part of their own commensal flora. Breaches of the skin or mucosal membrane allow S. aureus to initiate infection. Some diseases increase the risk of S. aureus infection; diabetes, for example, combines an increased rate of colonization and the use of injectable insulin with the possibility of impaired leukocyte function. Individuals with congenital or acquired qualitative or quantitative defects of polymorphonuclear leukocytes (PMNs) are at increased risk of S. aureus infections; this group includes neutropenic patients (e.g., those receiving chemotherapeutic agents), those with chronic granulomatous disease, and those with Job’s or Chédiak-Higashi syndrome. Other groups at risk include individuals with end-stage renal disease, HIV infection, skin abnormalities, or prosthetic devices.
++
++
++
A 19-year-old female from Guatemala presents to your office for a routine screening physical examination. At age 4 years she was diagnosed with acute rheumatic fever. She does not recall the specifics of her illness and remembers only that she was required to be on bed rest for 6 months. She has remained on penicillin V orally at a dose of 250 mg bid since that time. She asks if she can safely discontinue this medication. She has had only one other flare of her disease, at age 8, when she stopped taking penicillin at the time of her emigration to the United States. She is currently working as a day care provider. Her physical examination is notable for normal point of maximal impulse (PMI) with a grade III/VI holosystolic murmur that is heard best at the apex of the heart and radiates to the axilla. What do you advise the patient to do?
++
++
++
A. An echocardiogram should be performed to determine the extent of valvular damage before deciding if penicillin can be discontinued.
++
++
B. Penicillin prophylaxis can be discontinued because she has had no flares in 5 years.
++
++
C. She should change her dosing regimen to IM benzathine penicillin every 8 weeks.
++
++
D. She should continue on penicillin indefinitely as she had a previous recurrence, has presumed rheumatic heart disease, and is working in a field with high occupational exposure to Group A streptococcus.
++
++
E. She should replace penicillin prophylaxis with polyvalent pneumococcal vaccine every 5 years.
+
++
++
(Chap. 44) Recurrent episodes of rheumatic fever are most common in the first 5 years after the initial diagnosis. Penicillin prophylaxis is recommended for at least this period. After the first 5 years secondary prophylaxis is determined on an individual basis. Ongoing prophylaxis is currently recommended for patients who have had recurrent disease, have rheumatic heart disease, or work in occupations that have a high risk for reexposure to group A streptococcal infection. Prophylactic regimens are penicillin V, PO 250 mg bid, benzathine penicillin, 1.2 million units IM every 4 weeks, and sulfadiazine, 1 g PO daily. Polyvalent pneumococcal vaccine has no cross-reactivity with group A streptococcus.
++
++
++
Which of the following statements regarding enterococcal infections is true?
++
++
++
A. E. faecalis is the most frequently isolated enteroccal species in surgical site infections
++
++
B. Enterococci are the most common causative organisms in community-acquired endocarditis
++
++
C. Most enterococcal urinary fact infections occur de novo with no prior known underlying risk factor
++
++
D. Poor dentition and gingival disease is the usual proximal source for enterococcal bacteremia
++
++
E. Presence of enterococci in a sterily-obtained urine culture invariably indicates infection and warrants treatment.
+
++
++
(Chap. 45) Enterococci are well-known causes of nosocomial UTI—the most common infection caused by these organisms. Enterococcal UTIs are usually associated with indwelling catheters, instrumentation, or anatomic abnormalities of the genitourinary tract, and it is often challenging to differentiate between true infection and colonization (particularly in patients with chronic indwelling catheters). The presence of leukocytes in the urine in conjunction with systemic manifestations (e.g., fever) or local signs and symptoms of infection with no other explanation suggests the diagnosis. Enterococci are important causes of community- and health care-associated endocarditis, ranking second after staphylococci as an etiologic agent. Enterococcus faecalis is the most common species of enterococci in surgical site infections. Enterococcal species colonize the gut (not often the mouth) and thus most invasive infections are traced back to gut contamination, surgery, or invasive procedures.
++
++
++
Which of the following is the most common clinical presentation of acute rheumatic fever?
++
++
++
++
++
++
++
++
++
++
++
+
++
++
(Chap. 46) Acute rheumatic fever is almost universally due to group A streptococcal disease in present time, though virtually all streptococcal disease may be capable of precipitating rheumatic fever. Although skin infections may be associated with rheumatic fever, far and away the most common presentation is with preceding pharyngitis. There is a latent period of approximately 3 weeks from an episode of sore throat to presentation of acute rheumatic fever. The most common manifestations are fever and polyarthritis, with polyarthritis present in 60–75% of cases. Carditis may also be present, though somewhat less frequently in 50–60% of cases. Chorea and indolent carditis may have a subacute presentation. Chorea is present in 2–30% of affected individuals, while erythema marginatum and subcutaneous nodules are rare. Sixty percent of patients with acute rheumatic fever progress to rheumatic heart disease with the endocardium, pericardium, and myocardium all potentially involved. All patients with acute rheumatic fever should receive antibiotics sufficient to treat the precipitating group A streptococcal infection.
++
++
++
Which of the following statements regarding Corynebacterium diphtheria infections is true?
++
++
++
A. Alcoholism is a risk factor for adult diphtheria infections
++
++
B. Birds and horses provide animal reservoirs for C. diphtheria
++
++
C. Childhood vaccination imparts lifelong protective immunity to patients who receive the proper vaccination course
++
++
D. Cutaneous diphtheria is almost always caused by a toxigenic strain
++
++
E. Development of an effective vaccination has eliminated adult diphtheria infections in the United States
+
++
++
(Chap. 47) Immunity to diphtheria induced by childhood vaccination gradually decreases in adulthood. An estimated 30% of men 60–69 years old have antitoxin titers below the protective level. In addition to older age and lack of vaccination, risk factors for diphtheria outbreaks include alcoholism, low socioeconomic status, crowded living conditions, and Native American ethnic background. An outbreak of diphtheria in Seattle, Washington, between 1972 and 1982 comprised 1100 cases, most of which were cutaneous. Corynebacterium diphtheriae is transmitted via the aerosol route, usually during close contact with an infected person. There are no significant reservoirs other than humans. Cutaneous diphtheria is usually a secondary infection that follows a primary skin lesion due to trauma, allergy, or autoimmunity. Most often, these isolates lack the tox gene and thus do not express diphtheria toxin.
++
++
++
You are caring for a 34-year-old pregnant woman with fever, backache, myalgias, headache, and bacteremia. Blood cultures show listeria monocytogenes and she is promptly initiated on appropriate antibiotic therapy. What mode of transmission is most likely the source of her listeriosis?
++
++
++
A. Aerosol person-to-person transmission
++
++
++
++
C. Ingestion of contaminated food
++
++
++
++
+
++
++
(Chap. 48) L. monocytogenes usually enters the body via the gastrointestinal tract in foods. Listeriosis is most often sporadic, although outbreaks do occur. No epidemiologic or clinical evidence supports person-to-person transmission (other than vertical transmission from mother to fetus) or waterborne infection. In line with its survival and multiplication at refrigeration temperatures, L. monocytogenes is commonly found in processed and unprocessed foods of animal and plant origin, especially soft cheeses, delicatessen meats, hot dogs, milk, and cold salads; fresh fruits and vegetables can also transmit the organism. Because food supplies are increasingly centralized and normal hosts tolerate the organism well, outbreaks may not be immediately apparent. The U.S. Food and Drug Administration has a zero-tolerance policy for L. monocytogenes in ready-to-eat foods.
++
++
++
A 64-year-old man with a long history of heroin abuse is brought to the hospital because of fever and worsening muscle spasms and pain over the last day. Because of longstanding venous sclerosis, he no longer injects intravenously but ‘skin-pops’, often with dirty needles. On examination, he is extremely sweaty and febrile to 101.4° F. There are widespread muscle spasms including the face. He is unable to open his jaw due to muscle spasm and has severe back pain due to diffuse spasm. On his leg there is a skin wound that is tender and erythematous. All of the following statements regarding this patient are true EXCEPT:
++
++
++
A. Culture of the wound may reveal Clostridium tetani
++
++
B. Intrathecal antitoxin administration is recommended therapy
++
++
C. Metronidazole is recommended therapy
++
++
D. Permanent muscle dysfunction is likely after recovery
++
++
E. Strychnine poisoning and antidopaminergic drug toxicity should be ruled out
+
++
++
(Chap. 49) Tetanus is an acute disease manifested by skeletal muscle spasm and autonomic nervous system disturbance. It is caused by a powerful neurotoxin produced by the bacterium Clostridium tetani and now a rare disease due to widespread vaccination. There were <50 cases reported recently in the United States but a rising frequency in drug users. Older patients may be at higher risk due to waning immunity. The differential diagnosis of a patient presenting with tetanus includes strychnine poisoning and drug-related dystonic reactions. The diagnosis is clinical. Cardiovascular instability is common due to autonomic dysfunction and is manifest by rapid fluctuation in heart rate and blood pressure. Wound cultures will be positive in approximately 20% of cases. Metronidazole or penicillin should be administered to clear infection. Tetanus immune globulin is recommended over equine antiserum because of a lower risk of anaphylactic reactions. Recent evidence suggests that intrathecal administration is efficacious in inhibiting disease progression and improving outcomes. Muscle spasms may be treated with sedative drugs. With effective supportive care and often respiratory support muscle function recovers after clearing toxin with no residual damage.
++
++
++
A 6-month-old male infant is brought emergently to the hospital for evaluation of altered behavior and cyanosis. On examination, he is taking only shallow respirations and you note a flaccid muscle tone throughout. The parents note no preceding fever, cough, coryza, rash, or new medications. He had an uncomplicated pre- and ante-natal course and was growing and progressing normally through his cognitive and motor milestones prior to this. He recently started eating soft and pureed foods including yogurt, pureed broccoli, spinach, carrots, bananas with small amounts of added honey. He has had no scheduled vaccinations within the past month. Deep tendon reflexes are absent. Which of the following is the most likely cause of his presentation?
++
++
++
++
++
B. Guillain-Barré syndrome
++
++
C. Hypokalemic periodic paralysis
++
++
++
++
+
++
++
(Chap. 50) The clinical presentation is characteristic of botulinum toxicity. Botulinum toxicity results in a flaccid paralysis by inhibitor presynaptic acetylcholine release. Further, this infant has the history of honey ingestion which is a risk factor in infants. In adults, the ingestion of Clostridium botulinum spores does not lead to clinical botulism, as the pathogenic organisms do not readily grow and produce toxin in that environment. However, the risk is much higher for infants whose gut flora has not yet matured. For this reason, honey should not be given to infants <12 months of age. Tetanus toxicity causes a spastic, not flaccid, paralysis. Guillain-Barré syndrome has been reported in children, but is almost always associated with vaccinations or a preceding illness. The absence of mention of a tick found embedded in the skin makes tick paralysis unlikely. In hypokalemic periodic paralysis (a very rare inheritable cause of muscle weakness and paralysis due to a channelopathy) deep tendon reflexes are normal despite the weakness or paralysis.
++
++
++
You are a medical resident completing a clinical rotation in Papua New Guinea. Today, two adults present only hours apart to the emergency department with remarkably similar cases. Both had attended a pig feast yesterday, and awoke today with excruciating abdominal pain. Both CT scans show extensive necrosis of the intestinal wall, most severely in the jejunum. The causative organism of these infections is also most commonly causative in cases of which of the following?
++
++
++
++
++
B. Gas gangrene myonecrosis
++
++
++
++
++
++
+
++
++
(Chap. 51) Both of these patients have enteritis necroticans caused by Clostridium perfringens. In Papua New Guinea during the 1960s, enteritis necroticans (known in that locale as pigbel) was found to be the most common cause of death in childhood; it was associated with pig feasts and occurred both sporadically and in outbreaks. Intramuscular immunization against the β toxin resulted in a decreased incidence of the disease in Papua New Guinea, although the condition remains common. C. perfringens is also the cause of gas gangrene myonecrosis, a highly morbid and mortal condition. The most common cause of bacterial pneumonia is the pneumococcus. N. meningitidis and Streptococcus pneumonia are the most common causes of meningitis in adults. Group A streptococcus is the most common cause of bacterial pharyngitis, and staphylococcal and streptococcal species are the most common causes of cellulitis.
++
++
++
During your freshman year of college, you perform an experiment for your microbiology class. You obtain nasopharyngeal swab cultures on your roommates to analyze the colonizing bacteria there. You find that one of your roommates has evidence of Neisseria meningitidis from his culture. When he returns from class, he says he has been feeling fine. He does not think he has been vaccinated against meningococcus. The next most appropriate step is which of the following?
++
++
++
A. Administer ceftriaxone 1 g intramuscularly
++
++
B. Administer mupirocin cream to the bilateral nares twice daily
++
++
C. Administer quadrivalent meningococcal vaccine
++
++
D. Admit to the hospital for intravenous ceftriaxone 2 g daily for 14 days
++
++
+
++
++
(Chap. 52) A high proportion of young adults and adolescents (up to 25% in some studies) are colonized with N. meningiditis, likely due to lifestyle and high-risk activities (kissing, crowded living conditions). Changes in living conditions (e.g., freshman year of college) are associated with a higher risk of invasive infections. It is unlikely that treating noninvasive meningococcal colonization would be effective in reducing the risk of invasive disease given the likelihood of recolonization. However, the quadrivalent meningococcal vaccine is effective in preventing invasive meningococcal disease and is recommended for all children older than 11 years old.
++
++
++
Ms. Jones is a 27-year-old telephone repair technician who is 4 months pregnant. She is sexually active with her boyfriend. For the past 2 days, she has been experiencing dysuria and vaginal discharge. Gram stain of her vaginal discharge is shown in Figure 42. She has no known medication allergies. Which of the following is the most reasonable medical regimen for the treatment of her infection?
++
++
++
(From the Public Health Agency of Canada. © All rights reserved. Reproduced with permission from the Minister of Health, 2016.)
++
++
++
A. Ceftriaxone 250 mg IM once
++
++
B. Ceftriaxone 250 mg IM and azithromycin 1 g orally once
++
++
C. Ceftriaxone 250 mg IM and doxycycline 100 mg orally once
++
++
D. Ciprofloxacin 500 mg twice daily for 10 days
++
++
E. Vancomycin 1 g daily for 7 days
+
++
++
(Chap. 53) The Gram stain shows Gram-negative intracellular mono- and diplococci that, together with the clinical presentation, is diagnostic of the gonococcus. Highly effective single-dose regimens have been developed for uncomplicated gonococcal infections. Quinolone-containing regimens are no longer recommended in the United States as first-line treatment because of widespread resistance. Initial treatment regimens must also incorporate an agent (e.g., azithromycin or doxycycline) that is effective against chlamydial infection because of the high frequency of coinfection. Gonorrhea in pregnancy can have serious consequences for both the mother and the infant. Pregnant women with gonorrhea, who should not take doxycycline, should receive concurrent treatment with a macrolide antibiotic for possible chlamydial infection.
++
++
++
Which of the following is the most common bacterial cause of COPD exacerbations?
++
++
++
A. Streptococcus pneumonia
++
++
++
++
++
++
D. Nontypeable haemophilus influenza
++
++
+
++
++
(Chap. 54) While most COPD exacerbations are thought to be due to upper respiratory viral infection, some proportion are associated with culturable pathogenic bacteria. Nontypable Haemophilus influenzae is the most common bacterial cause of exacerbations of COPD; these exacerbations are characterized by increased cough, sputum production, and shortness of breath. Fever is low-grade, and no infiltrates are evident on chest x-ray. Moraxella catarrhalis, streptococcus pneumonia, and pseudomonas aeruginosa most commonly cause clinical pneumonias instead of COPD exacerbations. Acinetobacter can also cause pneumonia, though often in patients with intrinsic airway disease or alterations (e.g., bronchiectasis). Approximately 20–35% of nontypable strains produce β-lactamase (with the exact proportion depending on geographic location), and these strains are resistant to ampicillin. Several agents have excellent activity against nontypable H. influenzae, including amoxicillin/clavulanic acid, various extended-spectrum cephalosporins, macrolides (azithromycin, clarithromycin) and fluoroquinolones.
++
++
++
All of the following statements regarding the HACEK organisms are true EXCEPT:
++
++
++
A. HACEK-associated endocarditis tends to occur in younger patients than non-HACEK endocarditis
++
++
B. HACEK organisms require an oxygen-enriched environment for growth
++
++
C. Most cultures that ultimately yield a HACEK organism become positive within the first week
++
++
D. The most common clinical manifestation of the HACEK organisms is endocarditis
++
++
E. They are fastidious slow-growing organisms
+
++
++
(Chap. 55) HACEK organisms are a group of fastidious, slow-growing, Gram-negative bacteria whose growth requires an atmosphere of carbon dioxide. Species belonging to this group include several Haemophilus species, Aggregatibacter (formerly Actinobacillus) species, Cardiobacterium hominis, Eikenella corrodens, and Kingella kingae. HACEK bacteria normally reside in the oral cavity and have been associated with local infections in the mouth. They are also known to cause severe systemic infections—most often bacterial endocarditis, which can develop on either native or prosthetic valves. Compared with non-HACEK endocarditis, HACEK endocarditis occurs in younger patients and is more frequently associated with embolic, vascular, and immunologic manifestations but less commonly associated with congestive heart failure. The clinical course of HACEK endocarditis tends to be subacute, particularly with Aggregatibacter or Cardiobacterium. However, K. kingae endocarditis may have a more aggressive presentation. Systemic embolization is common. The microbiology laboratory should be alerted when a HACEK organism is being considered; however, most cultures that ultimately yield a HACEK organism become positive within the first week, especially with improved culture systems.
++
++
++
Legionella outbreaks are usually tied to which of the following?
++
++
++
A. A significant population of unvaccinated individuals
++
++
++
++
C. Contaminated aquatic reservoir
++
++
D. Equine animal reservoir and zoonotic transmission
++
++
E. Surgical wound infections
+
++
++
(Chap. 56) The natural habitats for Legionella pneumophila are aquatic bodies, including lakes and streams. Natural bodies of water contain only small numbers of legionellae. However, once the organisms enter human-constructed aquatic reservoirs (such as drinking-water systems), they can grow and proliferate. Factors known to enhance colonization by and amplification of legionellae include warm temperatures (25–42°C) and the presence of scale and sediment. L. pneumophila can form microcolonies within biofilms; its eradication from drinking-water systems requires disinfectants that can penetrate the biofilm. Heavy rainfall and flooding can result in the entry of high numbers of legionellae into water-distribution systems, leading to an upsurge of cases. Large buildings over three stories high are commonly colonized with Legionella. Sporadic community-acquired Legionnaires’ disease has been linked to colonization of hotels, office buildings, factories, and even private homes. Drinking-water systems in hospitals and extended-care facilities have been the source for health care-associated Legionnaires’ disease. Transmission is via inhalation of bacteria-containing aerosol. The disease is not spread via zoonotic or insect vectors nor person-to-person transmission. There is no effective vaccination for L. pneumophila.
++
++
++
Which of the following statements regarding the epidemiology of pertussis infection is true?
++
++
++
A. Completing the pertussis vaccination series confers lifelong immunity for immune-competent patients
++
++
B. In North America, pertussis infection rates are highest in summer and autumn
++
++
C. Pertussis infection rates have remained relatively stable year to year after the widespread adoption of vaccination
++
++
D. Pertussis is uniquely a disease of childhood—adults and adolescents suffer no symptoms from pertussis infection
++
++
E. Worldwide vaccination has made infant mortality from pertussis exceedingly rare globally
+
++
++
(Chap. 57) Pertussis is a highly communicable disease, with attack rates of 80–100% among unimmunized household contacts and 20% within households in well-immunized populations. The infection has a worldwide distribution, with cyclical outbreaks every 3–5 years (a pattern that has persisted despite widespread immunization). Pertussis occurs in all months; however, in North America, its activity peaks in summer and autumn. In developing countries, pertussis remains an important cause of infant morbidity and death. The reported incidence of pertussis worldwide has decreased as a result of improved vaccine coverage. However, coverage rates are still <50% in many developing nations; the World Health Organization (WHO) estimates that 90% of the burden of pertussis is in developing regions. Although thought of as a disease of childhood, pertussis can affect people of all ages and is increasingly being identified as a cause of prolonged coughing illness in adolescents and adults. The duration of immunity after whole-cell pertussis vaccination is short-lived, with little protection remaining after 10–12 years. Recent studies have demonstrated early waning of immunity—i.e., within 2–4 years after the fifth dose of vaccine in children who received the acellular pertussis vaccine for their primary series in infancy. These data suggest that boosters may be needed more frequently than every 10 years, as previously thought.
++
++
++
In healthy individuals, which of the following bacteria species is the predominant Gram-negative bacillus in the colonic flora?
++
++
++
++
++
++
++
++
++
++
++
+
++
++
(Chap. 58) In healthy humans, E. coli is the predominant species of Gram-negative bacilli (GNB) in the colonic flora; Klebsiella and Proteus spp. are less prevalent. Both staphylococcus and clostridium species are Gram-positive organisms.
++
++
++
A 63-year-old man has been in the ICU for 3 weeks with slowly resolving acute respiratory distress syndrome (ARDS) after an episode of acute pancreatitis. He remains on mechanical ventilation through a tracheostomy. Over the last week he has had gradual lessening of his mechanical ventilator needs and slight improvement of his radiograph. He has been afebrile with a normal WBC for the last 10 days. Over the last 24 hours his FiO2 has been increased from 0.60 to 0.80 to maintain adequate oxygenation. In addition he has developed newly purulent sputum with a right lower lobe infiltrate, fever to 101.5°C and a rising WBC. Sputum Gram stain shows Gram-negative plump coccobacilli that are identified as Acinetobacter baumannii. All of the following statements regarding infections due to this organism are true EXCEPT:
++
++
++
A. It is a growing cause of hospital acquired pneumonia and bloodstream infections in the United States
++
++
B. It is not yet a significant problem in Asia/Australia
++
++
C. Mortality from bloodstream infection approaches 40%
++
++
D. Multidrug resistance is characteristic
++
++
E. Tigecycline is treatment of choice for bloodstream infection
+
++
++
(Chap. 59) Infections with Acinetobacter spp. are a growing cause of hospital acquired infection worldwide. Surveillance data from Australia and Asia suggest that infections are common and there are reports of community acquired acinetobacter infection. They typically infect patients receiving long-term care in ICUs by causing ventilator associated pneumonia, bloodstream infections, or urinary tract infections. They are particularly of concern because of their propensity to develop multidrug (or pan-drug) resistance and their ability to colonize units due to health care worker transmission. Acinetobacter baumannii is the most common isolate and develops drug resistance avidly. Many strains are currently resistant to carbapenems (imipenem, meropenem). Last line agents such as colistin, polymyxin A, and tigecycline are often the only available therapeutic options. Tigecycline has been used for pneumonia due to carbapenem resistant strains but is not thought to be efficacious in bloodstream infection because usual dosing does not achieve therapeutic levels against acinetobacter.
++
++
++
A 42-year-old man with heme occult positive stools and a history of epigastric pain is found to have a duodenal ulcer that is biopsy proven positive for H. pylori. All of the following are effective eradication regimens EXCEPT:
++
++
++
A. Amoxicillin, levofloxacin for 10 days
++
++
B. Omeprazole, clarithromycin, metronidazole for 14 days
++
++
C. Omeprazole, clarithromycin, amoxicillin for 14 days
++
++
D. Omeprazole, bismuth, tetracycline, metronidazole for 14 days
++
++
E. Omeprazole, amoxicillin for 5 days followed by omeprazole, clarithromycin, tinidazole for 5 days
+
++
++
(Chap. 60) In vitro, H. pylori is susceptible to a wide variety of antibiotics. However, monotherapy is no longer recommended because of inadequate antibiotic delivery to the colonization niche and the development of resistance. All current regimens include either a proton pump inhibitor (omeprazole or equivalent), H2 blocker (ranitidine or equivalent), and/or bismuth. Regimens including quinolones may not be advisable because of common resistance and the risk of developing C. difficile colitis. Current regimens have an eradication rate of 75–80%.
++
++
++
A sputum culture from a patient with cystic fibrosis showing which of the following organisms has been associated with a rapid decline in pulmonary function and a poor clinical prognosis?
++
++
++
++
++
B. Pseudomonas aeruginosa
++
++
++
++
D. Staphylococcus epidermidis
++
++
E. Stenotrophomonas maltophilia
+
++
++
(Chap. 61) Burkholderia cepacia is an opportunistic pathogen that has been responsible for nosocomial outbreaks. It also colonizes and infects the lower respiratory tract of patients with cystic fibrosis, chronic granulomatous disease, and sickle cell disease. In patients with cystic fibrosis it portends a rapid decline in pulmonary function and a poor clinical prognosis. It also may cause a resistant necrotizing pneumonia. B. cepacia is often intrinsically resistant to a variety of antimicrobials, including many β-lactams and aminoglycosides. Trimethoprim- sulfamethoxazole (TMP/SMX) is usually the first-line treatment. P. aeruginosa and S. aureus are common colonizers and pathogens in patients with cystic fibrosis. Repeated infections due to these agents will result in lung function deterioration. However, airway colonization of B. cepacia has the strongest risk of declining lung function and worsening function. Stenotrophomonas maltophilia is an opportunistic pathogen, particularly in patients with cancer, transplants, and critical illness. S. maltophilia is a cause of pneumonia, urinary tract infection, wound infection, and bacteremia. TMP/SMX is usually the treatment of choice for Stenotrophomonas infections.
++
++
++
You are taking care of a 65-year-old zoo employee who presents with fever and diarrhea. Ultimately, you diagnose her with nontyphoidal salmonellosis. You know that she must have contracted the infection via what route?
++
++
++
A. Contact with contaminated fomites
++
++
B. Oral ingestion of organisms
++
++
C. Respiratory secretions
++
++
++
++
E. Any of the above routes
+
++
++
(Chap. 62) All Salmonella infections begin with ingestion of organisms, most commonly in contaminated food or water. The infectious dose ranges from 200 colony-forming units (CFU) to 106 CFU, and the ingested dose is an important determinant of incubation period and disease severity. Conditions that decrease either stomach acidity (an age of <1 year, antacid ingestion, or achlorhydric disease) or intestinal integrity (inflammatory bowel disease, prior gastrointestinal surgery, or alteration of the intestinal flora by antibiotic administration) increase susceptibility to Salmonella infection.
++
++
++
Two days after returning from a trip to Thailand, a 36-year-old woman develops severe crampy abdominal pain, fever to 40°C, nausea, and malaise. The next day she begins having bloody mucopurulent diarrhea with worsening abdominal pain and continued fever. She reports she was in Bangkok during monsoonal flooding and ate fresh food from stalls. A stool examination shows many neutrophils and culture grows Shigella flexneri. Which of the following statements regarding her clinical syndrome is true?
++
++
++
A. An effective vaccine for travelers is available
++
++
B. Antibiotic therapy prolongs the carrier state and should not be administered unless she develops bacteremia
++
++
C. Antimotility agents are effective in reducing the risk of dehydration
++
++
D. Ciprofloxacin is recommended therapy
++
++
E. Her disease can be distinguished from illness due to Campylobacter jejuni on clinical grounds by the presence of fever
+
++
++
(Chap. 63) Shigellosis remains a cause of dysentery in the developing world and sporadic cases due to fecal-oral contamination occur in the developing and developed world. The human intestinal tract is the most prevalent reservoir for the bacteria. Clinical illness due to Shigella can be caused by a very small inoculum. Shigellosis typically evolves through four phases: incubation, watery diarrhea, dysentery, and the post-infectious phase. The incubation period is usually 1–4 days and the dysentery follows within hours to days. The dysentery syndrome is indistinguishable from other invasive enteropathogens (including campylobacter) and inflammatory bowel disease is also in the differential. Because the organism is enteroinvasive, antibiotic therapy is indicated. Ciprofloxacin is generally recommended unless there is no or proven resistance. Ceftriaxone, azithromycin, pivmecillinam, and some recent quinolones are also effective. Shigella infection typically does not cause life-threatening dehydration. Antimotility agents are not recommended because they are thought to prolong the systemic symptoms and may increase the risk of toxic megacolon and hemolytic-uremic syndrome. There is currently no commercially available vaccine for Shigella infection.
++
++
++
A 45-year-old healthy businessman just returned from a trip to Vietnam where he was traveling on vacation. Five days after his return, he developed fever and a headache followed 12 hours later by diarrhea and abdominal pain. The next day, he had >10 bowel movements in 12 hours, the final two with gross blood in the stool, prompting his presentation to the emergency department. His vital signs are as follows: Temperature 37.8°C, heart rate 90 beats/min, respiratory rate 14/min, oxygen saturation 98% on room air. Gram stain of a stool specimen identifies a small curved helical shaped Gram-negative rod. You suspect a Campylobacter spp. Which of the following statements regarding his most likely infection is true?
++
++
++
A. A single dose of azithromycin is effective therapy
++
++
B. Antibiotics are not helpful. He should receive supportive fluid and electrolyte repletion
++
++
C. Ciprofloxacin for 7 days is the therapy of choice
++
++
D. If the Campylobacter subtype is jejuni, the patient should be monitored closely for systemic infection and distal organ involvement (seeding)
++
++
E. The campylobacter subtype fetus carries a more favorable prognosis than other subtypes
+
++
++
(Chap. 64) This patient is suffering from classic traveler’s diarrhea (of which, campylobacter is a common cause). A prodrome of fever, headache, myalgia, and/or malaise often occurs 12–48 hours before the onset of diarrheal symptoms. The most common signs and symptoms of the intestinal phase are diarrhea, abdominal pain, and fever. The degree of diarrhea varies from several loose stools to grossly bloody stools; most patients presenting or medical attention have ≥10 bowel movements on the worst day of illness. Abdominal pain usually consists of cramping and may be the most prominent symptom. Pain is usually generalized but may become localized; C. jejuni infection may cause pseudoappendicitis. Fever may be the only initial manifestation of C. jejuni infection, a situation mimicking the early stages of typhoid fever. Even among patients presenting for medical attention with Campylobacter enteritis, not all clearly benefit from specific antimicrobial therapy. Indications for therapy include high fever, bloody diarrhea, severe diarrhea, persistence for >1 week, and worsening of symptoms. Macrolides are generally the empirical treatment of choice with <10% of isolates demonstrating resistance. A single dose of azithromycin 500 mg is effective as is a 5- to 7-day course of erythromycin (250 mg orally four times daily or—for children—30–50 mg/kg per day, in divided doses) is the regimen of choice. Resistance to quinolones as well as to tetracyclines is substantial; ~22% of U.S. isolates in 2010 were resistant to ciprofloxacin. Except in infection with C. fetus, bacteremia is uncommon, developing most often in immunocompromised hosts and at the extremes of age. Due to its proclivity to cause bacteremia and distant organ involvement, Campylobacter fetus has a far worse prognosis than other subtypes. Systemic infection with C. fetus is much more often fatal than that due to related species; this higher mortality rate reflects in part the population affected.
++
++
++
While you are caring for villagers stricken with cholera in central Africa, you encounter Mr. Zi, a 22-year-old truck driver with cholera. He has the typical rice-water stools and has had 7 bowel movements today before noon. He is thirsty, but is able to hold a coherent conversation with you and can stand up without feeling light-headed. His heart rate is 87 beats/min, and blood pressure is 105/70 mmHg. What is the most appropriate treatment for Mr. Zi?
++
++
++
A. Erythromycin 250 mg orally four times daily for 3 days
++
++
B. Intravenous fluid replacement with normal saline, 100 mL/kg over 3 hours
++
++
C. Intravenous fluid replacement with Lactated Ringer’s solution, 100 mg/kg over 3 hours
++
++
D. Mix 0.5 teaspoon of table salt and 6 teaspoons of table sugar with 1 L of sterile water and have the patient drink up to 2 L of this solution daily
++
++
E. No treatment is necessary
+
++
++
(Chap. 65) Cholera can be a devastating and rapidly deadly disease, but some patients, such as Mr. Zi, are only mildly affected. The table below describes clinical characteristics of patients with mild, moderate, or severe dehydration due to cholera. In patients with mild dehydration, antibiotics are not needed as they are not curative. In patients with moderate or severe dehydration, the use of an antibiotic to which the organism is susceptible diminishes the duration and volume of fluid loss and hastens clearance of the organism from the stool. Erythromycin is the drug of choice. For patient with mild diarrhea from cholera, oral rehydration therapy is an excellent therapeutic option. ORS takes advantage of the hexose-Na+ cotransport mechanism to move Na+ across the gut mucosa together with an actively transported molecule such as glucose (or galactose). Cl– and water follow. This transport mechanism remains intact even when cholera toxin is active. ORS may be made by adding safe water to prepackaged sachets containing salts and sugar or by adding 0.5 teaspoon of table salt and 6 teaspoons of table sugar to 1 L of safe water. (See Table 65-1.)
++
++
++
++
Mr. Hou is a 56-year-old former pig farmer from China who has been in the United States for the past year visiting his sister. Shortly after arriving, he began experiencing fevers. Curiously, he has noted that while the fever has persisted for a year, it has followed the unusual undulating pattern. The fever would usually last for 2 weeks, and then relent for about 2 weeks at a time before returning. He also experienced joint pains and myalgias during his times of fever. More recently, he has experienced lower back pain with movement and at rest. He denies any weakness or numbness. A serologic investigation returns positive for IgG antibodies for Brucella. Which of the following statements regarding his condition is true?
++
++
++
A. A safe and effective human vaccine is available for Brucellosis
++
++
B. In this patient, Brucella melitensis is more likely than Brucella suis
++
++
C. Prior to starting treatment, coinfection with tuberculosis should be excluded
++
++
D. The presence of IgG antibodies is nonspecific. Given the atypical presentation, this likely is not a true Brucella infection
++
++
E. With appropriate treatment, the patient’s relapse rate from his disease is 1–2%
+
++
++
(Chap. 66) This patient has a classic presentation for chronic Brucellosis. In endemic areas, brucellosis may be difficult to distinguish from the many other causes of fever. The true global prevalence of human brucellosis is unknown because of the imprecision of diagnosis and the inadequacy of reporting and surveillance systems in many countries. Recently, there has been increased recognition of the high incidence of brucellosis in India and parts of China and of importations to countries in Oceania, such as Fiji. However, two features recognized in the 19th century distinguish brucellosis from other tropical fevers, such as typhoid and malaria: (1) Left untreated, the fever of brucellosis shows an undulating pattern that persists for weeks before the commencement of an afebrile period that may be followed by relapse. (2) The fever of brucellosis is associated with musculoskeletal symptoms and signs in about one-half of all patients. The clinical syndromes caused by the different species are similar, although B. melitensis tends to be associated with a more acute and aggressive presentation than B. suis. Vaccines based on live attenuated Brucella strains, such as B. abortus strain 19BA or 104M, have been used in some countries to protect high-risk populations but have displayed only short-term efficacy and high reactogenicity. The broad aims of antimicrobial therapy are to treat and relieve the symptoms of current infection and to prevent relapse. Focal disease presentations may require specific intervention in addition to more prolonged and tailored antibiotic therapy. In addition, tuberculosis must always be excluded, or—to prevent the emergence of resistance—therapy must be tailored to specifically exclude drugs active against tuberculosis (e.g., rifampin used alone) or to include a full antituberculous regimen. Serologic examination often provides the only positive laboratory findings in brucellosis. In acute infection, IgM antibodies appear early and are followed by IgG and IgA.
++
++
++
A 45-year-old man from western Kentucky presents to the emergency room in September complaining of fevers, headaches, and muscle pains. He recently had been on a camping trip with several friends during which they hunted for their food, including fish, squirrels, and rabbits. He did not recall any tick bites during the trip, but does recall having several mosquito bites. For the past week, he has had an ulceration on his right hand with redness and pain surrounding it. He also has noticed some pain and swelling near his right elbow. None of the friends he camped with have been similarly ill. His vital signs are: blood pressure 106/65 mmHg, heart rate 116 beats/min, respiratory rate 24/min, and temperature 38.7°C. His oxygen saturation is 93% on room air. He appears mildly tachypneic and flushed. His conjunctiva are not injected and his mucous membranes are dry. The chest examination reveals crackles in the right mid-lung field and left base. His heart rate is tachycardic but regular. There is a II/VI systolic ejection murmur heard best at the lower left sternal border. His abdominal examination is unremarkable. On the right hand, there is an erythematous ulcer with a punched-out center covered by a black eschar. He has no cervical lymphadenopathy, but there are markedly enlarged and tender lymph nodes in the right axillae and epitrochlear regions.
++
The epitrochlear node has some fluctuance with palpation. A chest x-ray shows fluffy bilateral alveolar infiltrates. Over the first 12 hours of his hospitalization, the patient becomes progressively hypotensive and hypoxic, requiring intubation and mechanical ventilation. What is the most appropriate therapy for this patient?
++
++
++
A. Ampicillin, 2 g IV q6h
++
++
B. Ceftriaxone, 1 g IV daily
++
++
C. Ciprofloxacin, 400 mg IV twice daily
++
++
D. Doxycycline, 100 mg IV twice daily
++
++
E. Gentamicin, 5 mg/kg twice daily
+
++
++
(Chap. 67) The most likely infecting organism in this patient is Francisella tularensis. Gentamicin is the antibiotic of choice for the treatment of tularemia. Fluoroquinolones have shown in vitro activity against F. tularensis and have successfully been used in a few cases of tularemia. Currently, however, it cannot be recommended as first-line therapy as data are limited in regards to its efficacy relative to gentamicin, but can be considered if an individual is unable to tolerate gentamicin. To date, there have been no clinical trials of fluoroquinolones to definitively demonstrate equivalency with gentamicin. Third-generation cephalosporins have in vitro activity against F. tularensis. However, use of ceftriaxone in children with tularemia resulted in almost universal failure. Likewise, tetracycline and chloramphenicol also have limited usefulness with a higher relapse rate (up to 20%) when compared to gentamicin. F. tularensis is a small Gram-negative, pleomorphic bacillus that is found both intra- and extracellularly. It is found in mud, water, and decaying animal carcasses, and ticks and wild rabbits are the source for most human infections in the southeast United States and Rocky Mountains. In western states, tabanid flies are the most common vectors. The organisms usually enter the skin through the bite of a tick or through an abrasion. On further questioning, the patient above reported that during the camping trip he was primarily responsible for skinning the animals and preparing dinner. He did suffer a small cut on his right hand at the site where the ulceration is apparent. The most common clinical manifestations of F. tularensis are ulceroglandular and glandular disease, accounting for 75–85% of cases. The ulcer appears at the site of entry of the bacteria and lasts for 1–3 weeks and may develop a black eschar at the base. The draining lymph nodes become enlarged and fluctuant. They may drain spontaneously. In a small percentage of patients, the disease becomes systemically spread, as is apparent in this case, with pneumonia, fevers, and sepsis syndrome. When this occurs, the mortality rate approaches 30% if untreated. However, with appropriate antibiotic therapy the prognosis is very good. Diagnosis requires a high clinical suspicion as demonstration of the organism is difficult. It rarely seen on Gram stain because the organisms stain weakly and are so small that they are difficult to distinguish from background material. On polychromatically stained tissue, they may be seen both intra- and extracellularly, singly or in clumps. Moreover, F. tularensis is a difficult organism to culture and requires cysteine-glucose–blood agar. However, most labs do not attempt to culture the organism because of the risk of infection in laboratory workers, requiring biosafety level 2 practices. Usually the diagnosis is confirmed by agglutination testing with titers >1:160 confirming diagnosis.
++
++
++
You are working as a physician in Northern New Mexico when a case of human infection with Yersinia pestis is reported in a nearby hospital. You know that this patient most likely contracted the disease through what method?
++
++
++
A. Consumption of undercooked beef, pork, or mutton
++
++
B. Direct handling of an infected small mammal
++
++
C. Direct bite from an infected wild carnivore
++
++
++
++
E. Person-to-person contact
+
++
++
(Chap. 68) Yersinia pestis is the etiologic bacterial agent in the plague, a systemic zoonosis. Ancient DNA studies have confirmed that the 14th century “Black Death” in Europe was Y. pestis infection. Patients can present with the bubonic, septicemic, or pneumonic form of the disease. It predominantly affects small rodents in rural areas of Africa, Asia, and the Americas and is usually transmitted to humans by an arthropod vector (the flea). The oriental rat flea Xenopsylla cheopis is the most efficient vector for transmission of plague among rats and onward to humans. Plague can also be rarely acquired through the handling of living or dead small mammals (e.g., rabbits, hares, and prairie dogs) or wild carnivores (e.g., wildcats, coyotes, or mountain lions). Dogs and cats may bring plague-infected fleas into the home, and infected cats may transmit plague directly to humans by the respiratory route. The last recorded case of person-to-person transmission in the United States occurred in 1925.
++
++
++
The Bartonella species have adapted to survive in animals by existing in what immunologically protected site within the body?
++
++
++
++
++
B. Central nervous system
++
++
++
++
++
++
+
++
++
(Chap. 69) Bartonella species are fastidious, facultative intracellular, low-growing, Gram-negative bacteria that cause a broad spectrum of diseases in humans. Bartonella infections are typically transmitted by vectors such as fleas, ticks, mosquitos, or sand flies. Most Bartonella species have successfully adapted to survival in specific domestic or wild mammals. Prolonged intraerythrocytic infection in these animals creates a niche where the bacteria are protected from both innate and adaptive immunity and which serves as a reservoir for human infections.
++
++
++
Mr. Awayab is a 54-year-old truck driver in the Dominican Republic. He presents to the local health department with a penile lesion (shown in Figure 59A). He denies fever or chills, but has had several unprotected sexual encounters in the past 6 months. You swab the lesion and stain the collected material with a rapid Giemsa stain, visualizing the cell in Figure 59B. You know that Mr. Awayab’s infection is caused by which organism?
++
++
++
++
++
++
++
++
++
++
++
++
C. Klebsiella granulomatis
++
++
++
++
+
++
++
(Chap. 70) This is a classic case of donovanosis or granuloma inguinale, caused by Klebsiella granulomatis. Donovanosis has an unusual geographic distribution that includes Papua New Guinea, parts of southern Africa, India, the Caribbean, French Guyana, Brazil, and aboriginal communities in Australia. A lesion starts as a papule or subcutaneous nodule that later ulcerates after trauma. The incubation period is uncertain, but experimental infections in humans indicate a duration of ~50 days. Four types of lesions have been described: (1) the classic ulcerogranulomatous lesion (shown), a beefy red ulcer that bleeds readily when touched; (2) a hypertrophic or verrucous ulcer with a raised irregular edge; (3) a necrotic, offensive-smelling ulcer causing tissue destruction; and (4) a sclerotic or cicatricial lesion with fibrous and scar tissue. The diagnosis is confirmed by microscopic identification of Donovan bodies (Figure 70-2) in tissue smears. Preparation of a good-quality smear is important. If donovanosis is suspected on clinical grounds, the smear for Donovan bodies should be taken before swab samples are collected to be tested for other causes of genital ulceration so that enough material can be collected from the ulcer. A swab should be rolled firmly over an ulcer previously cleaned with a dry swab to remove debris. Smears can be examined in a clinical setting by direct microscopy with a rapid Giemsa or Wright’s stain. Alternatively, a piece of granulation tissue crushed and spread between two slides can be used. Donovan bodies can be seen in large, mononuclear (Pund) cells as Gram-negative intracytoplasmic cysts filled with deeply staining bodies that may have a safety-pin appearance (shown in the image). Haemophilus ducreyi is the causative organism of chancroid which manifests in males as sexually transmitted painful genital ulcers.
++
++
++
Figure 70-2: Pund cell stained by rapid Giemsa (RapiDiff) technique. Numerous Donovan bodies are visible.
++
++
++
All of the following factors influence the likelihood of transmitting active tuberculosis EXCEPT:
++
++
++
A. Duration of contact with an infected person
++
++
B. Environment in which contact occurs
++
++
C. Presence of extrapulmonary tuberculosis
++
++
D. Presence of laryngeal tuberculosis
++
++
E. Probability of contact with an infectious person
+
++
++
(Chap. 74) Tuberculosis is most commonly transmitted from person to person by airborne droplets. Factors that affect likelihood of developing tuberculosis infection include the probability of contact with an infectious person, the intimacy and duration of contact, the degree of infectiousness of the contact, and the environment in which the contact takes place. The most infectious patients are those with cavitary pulmonary or laryngeal tuberculosis with about 105–107 tuberculous bacteria per milliliter of sputum. Individuals who have a negative AFB smear with a positive culture for tuberculosis are less infectious but may transmit the disease. However, individuals with only extrapulmonary (e.g., renal, skeletal) tuberculosis are considered noninfectious.
++
++
++
A 50-year-old man is admitted to the hospital for active pulmonary tuberculosis with a positive sputum acid-fast bacilli smear. He is HIV positive with a CD4 count of 85/μL and is not on highly active antiretroviral therapy. In addition to pulmonary disease, he is found to have disease in the L4 vertebral body. What is the most appropriate initial therapy?
++
++
++
A. Isoniazid, rifampin, ethambutol, and pyrazinamide
++
++
B. Isoniazid, rifampin, ethambutol, and pyrazinamide; initiate antiretroviral therapy
++
++
C. Isoniazid, rifampin, ethambutol, pyrazinamide, and streptomycin
++
++
D. Isoniazid, rifampin, and ethambutol
++
++
E. Withhold therapy until sensitivities are available
+
++
++
(Chap. 74) Initial treatment of active tuberculosis associated with HIV disease does not differ from that of a non-HIV infected person. The standard treatment regimen includes four drugs: isoniazid, rifampin, pyrazinamide, and ethambutol (RIPE). These drugs are given for a total of 2 months in combination with pyridoxine (vitamin B6) to prevent neurotoxicity from isoniazid. Following the initial 2 months, patients continue on isoniazid and rifampin to complete a total of 6 months of therapy. These recommendations are the same as those of non-HIV infected individuals. If the sputum culture remains positive for tuberculosis after 2 months, the total course of antimycobacterial therapy is increased from 6 to 9 months. If an individual is already on antiretroviral therapy (ART) at the time of diagnosis of tuberculosis, it may be continued, but often rifabutin is substituted for rifampin because of drug interactions between rifampin and protease inhibitors. In individuals not on ART at the time of diagnosis of tuberculosis, it is not recommended to start ART concurrently because of the risk of immune reconstitution inflammatory syndrome (IRIS) and an increased risk of medication side effects. IRIS occurs as the immune system improves with ART and causes an intense inflammatory reaction directed against the infecting organism(s). There have been fatal cases of IRIS in association with tuberculosis and initiation of ART. In addition, both ART and antituberculosis drugs have many side effects. It can be difficult for a clinician to decide which medication is the cause of the side effects and may lead unnecessarily to alterations in the antituberculosis regimen. ART should be initiated as soon as possible and preferably within 2 months. Three-drug regimens are associated with a higher relapse rate if used as a standard 6-month course of therapy and, if used, require a total of 9 months of therapy. Situations in which three-drug therapy may be used are pregnancy, intolerance to a specific drug, and in the setting of resistance. A five-drug regimen using RIPE plus streptomycin is recommended as the standard retreatment regimen. Streptomycin and pyrazinamide are discontinued after 2 months if susceptibility testing is unavailable. If susceptibility testing is available, the treatment should be based upon the susceptibility pattern. In no instance is it appropriate to withhold treatment in the setting of active tuberculosis to await susceptibility testing.
++
++
++
A 45-year-old patient with HIV/AIDS presents to the emergency department. He complains of a rash that has been slowly spreading up his right arm and is now evident on his chest and back. The rash consists of small nodules that have a reddish-blue appearance. Some of them are ulcerated, but there is minimal fluctuance or drainage. He is unsure when these began. He notes no foreign travel or unusual exposures. He is homeless and unemployed, but occasionally gets work as a day laborer doing landscaping and digging. A culture of a skin lesion grows a Mycobacterium in 5 days. Which of the following is the most likely organism?
++
++
++
++
++
++
++
++
++
++
++
+
++
++
(Chap. 76) Nontuberculous mycobacteria (NTM) were originally classified into “fast-growers” and “slow-growers” based on the length of time they took to grow in culture. While more sophisticated tests have been developed, this classification scheme is still used and is of some benefit to the clinician. Fast-growing NTM include M. abscessus, M. fortuitum, and M. chelonae. They will typically take 7 days or less to grow on standard media, allowing relatively fast identification and drug-resistance testing. Slow-growing NTM include M. avium, M. marinum, M. ulcerans, and M. kansasii. They often require special growth media and therefore a high pretest suspicion. The patient described above likely has a cutaneous infection from one of the “fast-growing” NTM, which could be diagnosed with tissue biopsy, Gram stain, and culture.
++
++
++
All of the following statements regarding antituberculosis therapeutic agents are true EXCEPT:
++
++
++
A. In the United States, M. tuberculosis resistance to isoniazid remains <10%
++
++
B. Optic neuritis is the most severe adverse effect of ethambutol
++
++
C. Pyrazinamide has utility in the therapy of M. avium complex and M. kansasii infections
++
++
D. Rifabutin should be used instead of rifampin in patients receiving concurrent treatment with protease inhibitors or nevirapine
++
++
E. Rifampin can decrease the half-life of warfarin, cyclosporine, prednisone, oral contraceptives, clarithromycin and other important drugs
+
++
++
(Chap. 77) Pyrazinamide (PZA) is first-line treatment for M. tuberculosis. Addition of PZA for 2 months to isoniazid and rifampin allows the total duration of treatment to be shortened from 9 months to 6 months. PZA has no utility in the treatment of nontuberculous mycobacteria. Ethambutol has no serious drug interactions but patients must be closely monitored for optic neuritis which may manifest with decreased visual acuity, central scotoma, or difficulty seeing green (or red). All patients initiating therapy with ethambutol should have a visual and ophthalmologic examination at baseline. In the United States overall, isoniazid resistance remains uncommon. Primary isoniazid resistance is more common in patients with tuberculosis born outside the United States. Rifampin is a potent inducer of cytochrome P450 system and has numerous drug interactions. The CDC has guidelines for managing antituberculosis drug interactions including rifampin (www.cdc.gov/tb/). Rifabutin is a less potent inducer of hepatic cytochromes. Rifabutin is recommended for HIV-infected patients who are on antiretroviral therapy with protease inhibitors or non-nucleoside reverse transcriptase inhibitors (particularly nevirapine) in place of rifampin.
++
++
++
A 68 year-old man is referred for evaluation of gait instability. He complains of lightning like pains lancinating to his thighs and last minutes. Examination is notable for impaired proprioceptive and vibratory sense in his feet. He has a wide-based, ataxic gait and a positive Romberg sign. His family reports that his wife had treated syphilis 35 years ago, but that he was never tested or treated for syphilis. Which ocular finding would be most consistent with neurosyphilis?
++
++
++
A. Pupils that react to accommodation but not light
++
++
B. Pupils that react to light but not accommodation
++
++
C. Pupils that react to both light and accommodation
++
++
D. Pupils that do not react to light or accommodation
+
++
++
(Chap. 78) Syphilitic involvement of the central nervous system can occur in both early or late syphilis and ranges from asymptomatic neurosyphilis, meningeal syphilis, meningovascular syphilis, general paresis and tabes dorsalis. While meningeal syphilis usually occurs within the first year after infection, general paresis or tabes dorsalis usually present after decades of latent infection. The primary ocular finding in both general paresis and in tabes dorsalis is the Argyll Robertson pupil. Argyll Robertson pupils fail to react to light but accommodation is preserved. Both pupils are typically involved. The pupils are typically small and do not dilate fully in dim light. However, the pupils constrict to near vision and dilate to far vision.
++
++
++
A 52-year-old man presents for evaluation of fever and jaundice. Flu-like symptoms including fever, chills, headache, and myalgias started suddenly 2 days ago. Yesterday he noted scleral icterus and darkened urine. Today he developed new cough, chest discomfort, dyspnea, and hemoptysis. He resides in Baltimore city and works as an exterminator. Two weeks ago he suffered a minor leg injury while abating a rat infestation in a dilapidated home also plagued with standing water. He has no other travel or exposure history. Which pattern of electrolytes are typical of acute kidney injury in the severe form of this spirochetal zoonosis?
++
++
++
A. Hyperkalemia and hyponatremia
++
++
B. Hyperkalemia and hypernatremia
++
++
C. Hypokalemia and hyponatremia
++
++
D. Hypokalemia and hypernatremia
+
++
++
(Chap. 80) The clinical triad of hemorrhage, jaundice, and acute kidney injury caused by pathogenic Leptospira infection is eponymously known as Weil’s syndrome. Leptospira affect most mammalian species but rats are an important reservoir. The organism may establish a symbiotic relationship with the host to persist in the urogenital tract for years. Water is a similarly important vehicle for the transmission of disease. Leptospirosis has been recognized in deteriorating inner cities with expanding rat populations. Disease ranges in severity from a mild illness that never comes to medical attention to rapidly progressive and severe illness with case-fatality rates as high as 50%. In the leptospiremic phase of illness, leptospires disseminate hematogenously to vital organs. Hemorrhagic complications may involve the lungs, GI tract, urogenital tract or skin and are often associated with thrombocytopenia. Jaundice is common but widespread hepatic necrosis is not. The kidneys are invariably involved. Renal involvement includes acute tubular damage and interstitial nephritis and a nonoliguric hypokalemic renal injury is characteristic of early leptospirosis. Deregulation of several transporters along the nephron (e.g., NKCC2, NHE1, Na/K ATPase, AQP1, AQP2) contribute to tubular electrolyte wasting and lead to hypokalemia and hyponatremia. Wasting of magnesium in the urine is also common in leptospiral nephropathy.
++
++
++
A 26-year-old man presents to your office complaining of recurrent episodes of fever and malaise. He returned from a camping trip in the northwestern part of Montana about 3 weeks ago. While he was hiking, he denies eating or drinking any unpasteurized milk products. He sterilized all of his water prior to drinking. He had multiple insect bites, but did not identify any ticks. He primarily slept in cabins or tents and did not notice any rodent droppings in the areas where he camped. Two friends that accompanied him on the trip have not been ill. He initially experienced fevers as high as 104.7°F (40.4°C) with myalgias, headache, nausea, vomiting, and diarrhea beginning 5 days after his return home. These symptoms lasted for about 3 days and resolved spontaneously. He attributed his symptoms to the “flu” and returned to his normal functioning. Seven days later, the fevers returned with temperatures to 105.1°F (40.6°C). With these episodes, his family noted him to have intermittent confusion. Today is day 4 of his current illness, and the patient feels that his fevers have again subsided. What is the most likely cause of the patient’s recurrent fevers?
++
++
++
++
++
++
++
++
++
D. Lymphocytic choriomeningitis
++
++
E. Tick borne relapsing fever
+
++
++
(Chap. 81) Tick borne relapsing fever (TBRF) is a spirochetal infection caused by any one of several species of Borrelia. The borreliae are small spirochetes that are transmitted to humans through the bite of an infected tick. The tick that transmits TBRF is Ornithodoros species, which feeds on a variety of squirrels and chipmunks that live near freshwater lakes. TBRF is endemic in several areas of the western United States, southern British Columbia, the Mediterranean, Africa, and the plateau regions of Mexico and South and Central America. In the United States, TBRF is rarely reported east of Montana, Colorado, New Mexico, and Texas. The general areas where TBRF is contracted are in the forested and mountainous regions of these states although it can be contracted in the limestone caves of central Texas. Only 13 counties in the entire United States have had 50% of all cases reported in the United States. After an incubation period of about 7 days, an individual infected with TBRF will begin to experience fevers that can reach as high as 106.7°F (41.5°C). Symptoms that accompany the fevers include myalgias, chills, nausea, vomiting, abdominal pain, confusion, and arthralgias. The average duration of a first episode is 3 days. If the disease is not recognized and treated, the fever will recur after a period of about 7 days. The duration of fevers is typically shorter with repeated episodes, but will continue to relapse about every 7 days until the disease is treated. Diagnosis of TBRF requires detection of the spirochetes in the blood during a febrile episode or serologic conversion. TBRF is typically treated with doxycycline or erythromycin for 7–10 days. The other options should be on the differential diagnosis for an individual with recurrent and relapsing fevers. In addition, this list would also include yellow fever, dengue fever, malaria, rat-bite fever, and infection with echovirus 9 or Bartonella species. Brucellosis is a bacterial infection most commonly transmitted by ingestion of contaminated milk or cheese, which this patient did not report. Colorado tick fever is a viral infection transmitted by the bite of a Dermacentor andersoni tick that is endemic in the western areas of the United States. The pattern of fever is slightly different from TBRF as the cycle is 2–3 days of fever followed by 2–3 days of normal temperature. Leptospirosis often has two phases of fever. The first occurs during the acute infection, lasting 7–10 days. In some individuals, the fever will recur 3–10 days later during the immune phase. The typical route of infection is prolonged contact with infected rodent droppings in wet environments. Lymphocytic choriomeningitis is a viral infection that is most commonly transmitted via contact with urine or droppings from the common house mouse. This illness usually has two phases as well. During the first phase that occurs 8–13 days after exposure, an individual will experience fevers, malaise and myalgias. In the second phase of illness, symptoms more typical of meningitis occur.
++
++
++
A 36-year-old man presents to the emergency department in Pennsylvania complaining of lightheadedness and dizziness. On physical examination, the patient is found to have a heart rate of 38 beats/min, and the ECG demonstrates acute heart block. On further questioning, he reports that he lives in a wooded area. He has two dogs that often roam in the woods and have been found with ticks on many occasions. He takes no medications and is otherwise healthy. He is an avid hiker and is also training for a triathlon. He denies any significant childhood illness. His family history is positive for an acute myocardial infarction in his father at age 42. His physical examination is normal with the exception of the slow but regular heartbeat. His chemistry panel shows no abnormalities. His chest radiograph is normal. What is the most likely cause of complete heart block in this individual?
++
++
++
A. Acute myocardial infarction
++
++
++
++
++
++
++
++
E. Subacute bacterial endocarditis
+
++
++
(Chap. 82) About 8% of individuals affected by Lyme disease have cardiac involvement during the second stage of disease. Caused by Borrelia burgdorferi, Lyme disease is transmitted by the bite of an infected Ixodes tick. The first phase of the disease represents localized infection and is characterized by the presence of the erythema migrans rash. The second stage of the disease represents disseminated infection. The most common manifestations of this stage are new annular skin lesions, headache, fever, and migratory arthralgias. When cardiac involvement is present, the most common presentation is related to conduction abnormalities, including all categories of heart block. Diffuse cardiac involvement can occur with acute myopericarditis and left ventricular dysfunction. The cardiac involvement typically resolves within a few weeks, even without treatment. Acute myocardial infarction can cause complete heart block, particularly in the event of an inferior MI. However, this patient has minimal risk factors for cardiac disease, is otherwise healthy, and has no symptoms to suggest this as a cause. Chagas disease is caused Trypanosoma cruzi, a parasite endemic to Mexico, Central and South America. Sarcoidosis is a systemic disease that pathologically demonstrates the diffuse presence of noncaseating granulomas in a variety of tissues. Conduction abnormalities including complete heart block and ventricular tachycardia can be the presenting symptoms of the disease. More commonly, sarcoidosis would have pulmonary manifestations. While sarcoidosis is certainly possible in this gentleman, it would be a diagnosis of exclusion as his risk factors make Lyme disease more likely. Subacute bacterial endocarditis can also result in complete heart block if the endocarditis progresses to develop a valve ring abscess. The patient with subacute bacterial endocarditis would present with a more acute illness than this gentleman with fevers, weight loss, and most likely secondary signs of endocarditis such as Osler nodes, splinter hemorrhages, and Janeway lesions.
++
++
++
A 27-year-old man who lives in North Carolina presents to his primary care physician complaining of fever, headache, myalgias, nausea, and anorexia 7 days after returning from hiking on the Appalachian Trail. Physical examination is remarkable for a temperature of 101.5°F (38.6°C). He appears generally fatigued but not toxic and does not have a rash. He is reassured by her primary care physician that this likely represents a viral illness, but returns to clinic 3 days later with a progressive rash and ongoing fevers. He states that small red spots began to appear on the wrists and ankles within 24 hours of the prior visit and have now progressed up his extremities and onto the trunk (see Figures 68A and B).
++
He also is noting increasing headache, and his wife thinks she has had some confusion. On physical examination, the patient is noted to be lethargic and answers questions slowly. What would be a reasonable course of action?
++
++
++
(Photos courtesy of Dr. Lindsey Baden; with permission.)
++
++
++
++
(Photos courtesy of Dr. Lindsey Baden; with permission.)
++
++
++
A. Admit the patient to the hospital for treatment with intravenous ceftriaxone 1 g twice daily and vancomycin 1 g twice daily
++
++
B. Admit the patient to the hospital for treatment with doxycycline 100 mg twice daily
++
++
C. Initiate treatment with doxycycline 100 mg orally twice daily as an outpatient
++
++
D. Initiate treatment with trimethoprim-sulfamethoxazole DS twice daily
++
++
E. Order rickettsial serologies and withhold treatment until a firm diagnosis is made
+
++
++
(Chap. 83) This patient demonstrates evidence of Rocky Mountain spotted fever (RMSF) which has progressed over the course of several days due to lack of initial recognition and treatment. RMSF is caused by infection with Rickettsia rickettsii and is transmitted through the bite of an infected dog tick. RMSF has been diagnosed in 47 states and is most commonly diagnosed in the south-central and southeastern states. Symptoms typically begin about 1 week following inoculation. The initial symptoms are vague and are easily misdiagnosed as a viral infection with fever, myalgias, malaise, and headache predominating. While almost all patients with RMSF develop a rash during the course of the illness, rash is present in only 14% on the first day, and the lack of rash in a patient who is at risk for RMSF should not delay treatment. By day 3, 49% of individuals develop a rash. The rash initially is a macular rash that begins on the wrists and ankles and progresses to involve the extremities and trunk. Over time, hemorrhaging into the macules occurs and has a petechial appearance. As the illness progresses, respiratory failure and CNS manifestations can develop. Encephalitis, presenting as confusion and lethargy, is present about 25% of the time. Other manifestations can include renal failure, hepatic injury, and anemia. Treatment for RMSF is doxycycline 100 mg twice daily. It can be administered orally or intravenously. As this patient shows progressive disease with CNS involvement, hospital admission for treatment is warranted to monitor for further decompensation in the patient’s condition. If the patient were more clinically stable, outpatient therapy would be appropriate. Treatment should not be delayed while awaiting confirmatory serologic testing as untreated cases of RMSF are fatal, usually within 8–15 days. Treatment with any sulfa drugs should be avoided as the drug is ineffective and can worsen the disease course. Intravenous ceftriaxone and vancomycin are appropriate agents for bacterial meningitis. Although this could be a consideration in this patient with fever, confusion, and a rash, meningococcemia would present with a more fulminant course, and the patient’s risk factor (hiking in an endemic area) would make RMSF more likely.
++
++
++
A previously healthy 19-year-old man presents with several days of headache, cough with scant sputum, dyspnea, and fever of 38.6°C. On examination, pharyngeal erythema is noted and lung fields showed scattered wheezes and some crackles. Chest radiograph shows bilateral peribronchial interstitial infiltrates. His hematocrit is 24.7%, down from a baseline measure of 46%. The only other laboratory abnormality is an indirect bilirubin of 3.4. A peripheral smear reveals no abnormalities. A cold agglutinin titer is measured at 1:64. What is the most likely infectious agent?
++
++
++
++
++
B. Legionella pneumophila
++
++
C. Methicillin-resistant Staphylococcus aureus
++
++
++
++
E. Streptococcus pneumoniae
+
++
++
(Chap. 84) Mycoplasma pneumoniae is a common cause of pneumonia that is often occurs in young previously healthy individuals and is often treated empirically with standard antibiotic regimens for community-acquired pneumonia. It is spread easily person-to-person, and outbreaks in crowded conditions, such as schools or barracks, are common. Most patients develop a cough without radiographic abnormalities. When radiographic abnormalities are present there is usually a diffuse bronchopneumonia pattern without any lobar consolidation. Pharyngitis and rhinitis are also common. M. pneumoniae commonly induces the production of cold agglutinins, which in turn can cause an IgM- and complement-mediated intravascular hemolytic anemia. The presence of cold agglutinins is specific for M. pneumoniae infection only in the context of a consistent clinical picture for infection, as in this patient. Cold agglutinins are more common in children. Measurement of cold agglutinins as a diagnostic test is mostly of historical interest and is not currently recommended since the development of respiratory secretion PCR testing. Blood smear shows no abnormality, which is in contrast to IgG or warm-type hemolytic anemia where spherocytes are seen.
++
++
++
A 20-year-old female is 36 weeks pregnant and presents for her first evaluation. She is diagnosed with Chlamydia trachomatis infection of the cervix. Upon delivery, what complication is her infant most at risk for?
++
++
++
++
++
++
++
++
++
++
++
E. Sensorineural deafness
+
++
++
(Chap. 85) Congenital infection from maternal transmission can lead to severe consequences for the neonate; thus, prenatal care and screening for infection are very important. C. trachomatis is associated with up to 25% of exposed neonates who develop inclusion conjunctivitis. It can also be associated with pneumonia and otitis media in the newborn. Pneumonia in the newborn has been associated with later development of bronchitis and asthma. Hydrocephalus can be associated with toxoplasmosis. Hutchinson triad, which is Hutchinson teeth (blunted upper incisors), interstitial keratitis, and eighth nerve deafness, is due to congenital syphilis. Sensorineural deafness can be associated with congenital rubella exposure. Treatment of C. trachomatis in the infant consists of oral erythromycin.
++
++
++
A 23-year-old woman is newly diagnosed with genital herpes simplex virus (HSV)-2 infection. What can you tell her that the chance of reactivation disease will be during the first year after infection?
++
++
++
++
++
++
++
++
++
++
++
+
++
++
(Chap. 88) Primary genital herpes due to HSV-2 is characterized by fever, headache, malaise, inguinal lymphadenopathy, and diffuse genital lesions of varying stage. The cervix and urethra are usually involved in women. While both HSV-2 and HSV-1 can involve the genitals, the recurrence rate of HSV-2 is much higher (90% in the first year) than with HSV-1 (55% in the first year). The rate of reactivation for HSV-2 is very high. Acyclovir, valacyclovir, and famciclovir are effective in shortening the duration of symptoms and lesions in genital herpes. Chronic daily therapy can reduce the frequency of recurrences in those with frequent reactivation. Valacyclovir has been shown to reduce transmission of HSV-2 between sexual partners.
++
++
++
A 65-year-old man is brought to the hospital by his wife because of new onset fever and confusion. He was well until 3 days ago, then has developed high fever, somnolence, and progressive confusion. His current medical history is unremarkable except for elevated cholesterol and his only medication is atorvastatin. He is a civil engineer at an international construction company. His wife reports he obtains regular health screening and has always been PPD negative. On admission, his temperature is 40°C and his vital signs are otherwise normal. He is confused and hallucinating. Soon after admission, he develops a tonic-clonic seizure that requires lorazepam to terminate. His head CT shows no acute bleeding or elevated ICP. An EEG shows an epileptiform focus in the left temporal lobe and diffusion weighted MRI shows bilateral temporal lobe inflammation. Which of the following is most likely to be diagnostic?
++
++
++
A. CSF acid fast staining
++
++
++
++
C. CSF PCR for herpes virus
++
++
D. CSF oligoclonal band testing
++
++
E. Serum cryptococcal antigen testing
+
++
++
(Chap. 88) Herpes encephalitis accounts for 10–20% of sporadic cases of viral encephalitis in the United States. It most commonly occurs in patients 5–30 and >50 years old. HSV-1 accounts for >95% of cases and most adults have clinical or serologic evidence of HSV-1 mucocutaneous infection before onset of CNS symptoms. HSV encephalitis is characterized by the acute onset of fever and focal neurologic signs, particularly in the temporal lobe. EEG abnormalities in the temporal lobe are common. CSF will show elevated protein, lymphocyte leukocytosis with red cells, and normal glucose. HSV PCR testing of CSF is highly sensitive and specific for diagnosis. Treatment with acyclovir reduces mortality; however, neurologic sequelae are common particularly in older patients. Differentiation of HSV encephalitis from other viral forms is difficult. Most experts recommend initiation of empiric acyclovir in any patient with suspected encephalitis pending a confirmed or alternative diagnosis. TB meningitis presents typically as a basilar meningitis and not encephalitis. The history, clinical findings, EEG abnormalities, and radiologic findings make fungal meningitis unlikely. CSF oligoclonal bands are typically seen in patient with multiple sclerosis.
++
++
++
Which of the following statements regarding administration of varicella zoster vaccine to patients >60 years old is true:
++
++
++
A. It is a killed virus vaccine so is safe in immunocompromised patients
++
++
B. It is not recommended for patients in this age group
++
++
C. It will decrease the risk of developing postherpetic neuralgia
++
++
D. It will not decrease the risk of developing shingles
++
++
E. It will not decrease the burden of disease
+
++
++
(Chap. 89) The currently available varicella zoster virus vaccine with 18 times the viral content of the live attenuated virus vaccine used in children was shown efficacious for shingles in patients >60 years of age. The vaccine decreased the incidence of shingles by 51%, the burden of illness by 61%, and the incidence of postherpetic neuralgia by 66%. The Advisory Committee on Immunization Practices has therefore recommended that persons in this age group be offered this vaccine in order to reduce the frequency of shingles and the severity of postherpetic neuralgia. Because is it a live virus vaccine, it should not be used in immunocompromised patients.
++
++
++
A 19-year-old college student comes to clinic reporting that he has been ill for 2 weeks. About 2 weeks ago, he developed notable fatigue and malaise that prevented him from his usual exercise regimen and caused him to miss some classes. Last week, he developed low grade fevers, sore throat, and swollen lymph nodes in his neck. He has a history of strep pharyngitis, so 3 days ago he took some ampicillin that he had in his possession. Over the last 2 days he has developed a worsening slightly itchy rash as shown in Figure 74.
++
++
++
(Courtesy of Maria Turner, MD; with permission.)
++
His physical examination is notable for a temperature of 38.1°C, pharyngeal erythema, bilateral tonsilar enlargement without exudates, bilateral tender cervical adenopathy, and a palpable spleen. All of the following statements regarding his illness are true EXCEPT:
++
++
++
A. More than 10% atypical lymphocytosis is likely
++
++
B. Heterophile antibody testing will likely be diagnostic
++
++
C. If heterophile antibody testing is negative, testing for IgG antibodies against viral capsid antigen will likely be diagnostic
++
++
D. It is spread via contaminated saliva
++
++
E. The patient can receive ampicillin in the future if indicated
+
++
++
(Chap. 90) Epstein-Barr virus (EBV) is the cause of heterophile-positive infectious mononucleosis (IM), which is characterized by fever, sore throat, lymphadenopathy, and atypical lymphocytosis. EBV is also associated with several human tumors, including nasopharyngeal carcinoma, Burkitt’s lymphoma, Hodgkin’s disease, and (in patients with immunodeficiencies) B cell lymphoma. EBV infection occurs worldwide with >90% of adults seropositive. In the developing world, most are infected as young children and IM is uncommon; whereas, in the more developed world most are infected as adolescents or young adults and IM is more common. The virus is spread by contaminated saliva. Asymptomatic seropositive individuals shed the virus in saliva. In young children, the EBV infection causes mild disease with sore throat. Adolescents and young adults develop IM as described above plus often splenomegaly in the second to third week of disease. The white blood cell count is usually elevated and peaks at 10,000–20,000/L during the second or third week of illness. Lymphocytosis is usually demonstrable, with >10% atypical lymphocytes. A morbilliform rash may occur in about 5% of patients as part of the acute illness. Most patients treated with ampicillin develop a macular rash as pictured; this rash is not predictive of future adverse reactions to penicillins. Heterophile antibody testing will be positive in up to 40% of cases of IM in the first week of illness and up to 90% by the third week. If heterophile antibody testing is negative, the more expensive testing for IgM antibodies to viral capsid antigen is more sensitive and specific. IgG antibodies to viral capsid antigen will stay present indefinitely after initial infection and are not useful for diagnosing acute disease. Treatment of uncomplicated IM is with rest, supportive measures, and reassurance. Excessive physical activity should be avoided in the first month to avoid splenic trauma. Prednisone is not indicated and may predispose to secondary infection. It has been used at high dose when IM is complicated by airway compromise due to pharyngeal swelling, autoimmune hemolytic anemia, severe thrombocytopenia, hemophagocytic syndrome, or other severe complications. Controlled trials have shown that acyclovir has no significant impact on the course of uncomplicated IM. One study showed no benefit for combined prednisone plus acyclovir.
++
++
++
In the patient described above, which of the following is indicated treatment?
++
++
++
++
++
B. Acyclovir plus prednisone
++
++
++
++
++
++
E. Rest, supportive measures, and reassurance
+
++
++
(Chap. 90) Epstein-Barr virus (EBV) is the cause of heterophile-positive infectious mononucleosis (IM), which is characterized by fever, sore throat, lymphadenopathy, and atypical lymphocytosis. EBV is also associated with several human tumors, including nasopharyngeal carcinoma, Burkitt’s lymphoma, Hodgkin’s disease, and (in patients with immunodeficiencies) B cell lymphoma. EBV infection occurs worldwide with >90% of adults seropositive. In the developing world, most are infected as young children and IM is uncommon; whereas, in the more developed world most are infected as adolescents or young adults and IM is more common. The virus is spread by contaminated saliva. Asymptomatic seropositive individuals shed the virus in saliva. In young children, the EBV infection causes mild disease with sore throat. Adolescents and young adults develop IM as described above plus often splenomegaly in the second to third week of disease. The white blood cell count is usually elevated and peaks at 10,000–20,000/L during the second or third week of illness. Lymphocytosis is usually demonstrable, with >10% atypical lymphocytes. A morbilliform rash may occur in about 5% of patients as part of the acute illness. Most patients treated with ampicillin develop a macular rash as pictured; this rash is not predictive of future adverse reactions to penicillins. Heterophile antibody testing will be positive in up to 40% of cases of IM in the first week of illness and up to 90% by the third week. If heterophile antibody testing is negative, the more expensive testing for IgM antibodies to viral capsid antigen is more sensitive and specific. IgG antibodies to viral capsid antigen will stay present indefinitely after initial infection and are not useful for diagnosing acute disease. Treatment of uncomplicated IM is with rest, supportive measures, and reassurance. Excessive physical activity should be avoided in the first month to avoid splenic trauma. Prednisone is not indicated and may predispose to secondary infection. It has been used at high dose when IM is complicated by airway compromise due to pharyngeal swelling, autoimmune hemolytic anemia, severe thrombocytopenia, hemophagocytic syndrome, or other severe complications. Controlled trials have shown that acyclovir has no significant impact on the course of uncomplicated IM. One study showed no benefit for combined prednisone plus acyclovir.
++
++
++
Which of the following manifestations of cytomegalovirus (CMV) infection is least likely to occur following lung transplantation?
++
++
++
A. Bronchiolitis obliterans
++
++
++
++
++
++
++
++
E. CMV syndrome (fever, malaise, cytopenias, transaminitis, and CMV viremia)
+
++
++
(Chap. 91) CMV retinitis, a common CMV infection in HIV patients, occurs less commonly in solid organ transplant patients. CMV does affect the lung in a majority of transplant patients if either donor or recipient is CMV-seropositive pretransplant. CMV disease in transplant recipients typically develops 30–90 days post-transplant. It rarely occurs within 2 weeks of transplantation. CMV very commonly causes a pneumonitis that clinically is difficult to distinguish from acute rejection. Prior CMV infection has been associated with bronchiolitis obliterans syndrome (chronic rejection) in lung transplant recipients. As with HIV, the gastrointestinal tract is commonly involved with CMV infection. Endoscopy with biopsy showing characteristic giant cells, not serum polymerase chain reaction (PCR), is necessary to make this diagnosis. The CMV syndrome is also common in lung transplant patients. Serum CMV PCR should be sent as part of the work up for all nonspecific fevers, worsening lung function, liver function abnormalities, or falling leukocyte counts occurring more than a couple of weeks after transplant.
++
++
++
Which of the following serology patterns places a transplant recipient at the lowest risk of developing cytomegalovirus (CMV) infection after renal transplantation?
++
++
++
A. Donor CMV IgG negative, recipient CMV IgG negative
++
++
B. Donor CMV IgG negative, recipient CMV IgG positive
++
++
C. Donor CMV IgG positive, recipient CMV IgG negative
++
++
D. Donor CMV IgG positive, recipient CMV IgG positive
++
++
E. The risk is equal regardless of serology results
+
++
++
(Chap. 91) When the transplant donor is CMV IgG positive and recipient is negative, there is a very high risk of primary CMV infection in the recipient. However, if the recipient is IgG positive, CMV occurs as a reactivation infection. When both donor and recipient are seronegative, then the risk of any CMV infection is lowest, but not zero, as a contact with an infected host could prompt primary CMV infection. Unlike nearly all other transplant patients, many donor and recipient seronegative patients do not receive chemoprophylaxis with ganciclovir. In patients who are CMV IgG negative and received a CMV IgG negative transplant, transfusions should be from CMV IgG negative donors or white blood cell filtered products administered to reduce the risk of primary CMV infection. It is not clear whether universal prophylaxis or preemptive therapy is the preferable approach in CMV-seropositive immunocompromised hosts. Both ganciclovir and valganciclovir have been used successfully for prophylaxis and preemptive therapy in transplant recipients. A CMV glycoprotein B vaccine reduced infections in a placebo-controlled trial among 464 CMV-seronegative women; this outcome raises the possibility that this experimental vaccine will reduce congenital infections, but further studies must validate this approach.
++
++
++
A 22-year-old woman presents with diffuse arthralgias and morning stiffness in her hands, knees, and wrists. Two weeks earlier she had a self-limited febrile illness notable for a red facial rash and lacy reticular rash on her extremities. On examination, her bilateral wrists, metacarpophalangeal joints, and proximal interphalangeal joints are warm and slightly boggy. Which of the following tests is most likely to reveal her diagnosis?
++
++
++
++
++
B. Chlamydia trachomatis ligase chain reaction of the urine
++
++
C. Joint aspiration for crystals and culture
++
++
++
++
+
++
++
(Chap. 93) The most likely diagnosis based on her antecedent illness with a facial rash is parvovirus infection. Arthropathy is uncommon in childhood parvovirus infection but may cause a diffuse symmetric arthritis in up to 50% of adults. This corresponds to the immune phase of illness when IgM antibodies are developed. The arthropathy syndrome is more common in women than men. The distribution of affected joints is typically symmetric most commonly in the small joints of the hands and less commonly the ankles, knees, and wrists. Occasionally the arthritis persists over months and can mimic rheumatoid arthritis. Rheumatoid factor can be detected in serum. Parvovirus B19V infection may trigger rheumatoid disease in some patients and has been associated with juvenile idiopathic arthritis. Reactive arthritis due to Chlamydia or a list of other bacterial pathogens tends to effect large joints such as the sacroiliac joints and spine. It is also sometimes accompanied by uveitis and urethritis. The large number of joints involved with a symmetric distribution argues against crystal or septic arthropathy.
++
++
++
Which of the following statements regarding the currently licensed human papillomavirus (HPV) vaccines is true?
++
++
++
A. Both protect against genital warts
++
++
B. Once sexually active, women will derive little protective benefit from vaccination
++
++
C. They are inactivated live virus vaccines
++
++
D. They are targeted toward all oncogenic strains of HPV but are only 70% effective at decreasing infection in an individual
++
++
E. Vaccinees should continue to receive standard Pap smear testing
+
++
++
(Chap. 94) Currently available HPV vaccines dramatically reduce rates of infection and disease produced by the HPV types in the vaccines. These products are directed against virus types that cause anogenital tract disease. Both vaccines consist of virus-like particles without any viral nucleic acid, therefore are not active. To date, one quadrivalent product (Gardasil, Merck) containing HPV types 6, 11, 16, and 18 and one bivalent product (Cervarix, GlaxoSmithKline) containing HPV types 16 and 18 have been licensed in the United States. HPV types 6 and 11 cause 90% of anogenital warts, whereas types 16 and 18 are responsible for 70% of cervical cancers. Efficacy has varied according to the immunologic and virologic characteristics of study populations at baseline and according to the endpoints evaluated. Among study participants who are shown at baseline not to be infected with a specific virus type contained in the vaccine and who adhere to the study protocol, rates of vaccine efficacy regularly exceed 90%, as measured by both infection and disease caused by that specific virus type. Study participants who are already infected at baseline with a specific virus type contained in the vaccine do not benefit from vaccination against that type but may benefit from vaccination against other virus types contained in the vaccine preparation. Thus available HPV vaccines have potent prophylactic effects but no therapeutic effects. The CDC’s Advisory Committee for Immunization Practice recommends administration of the quadrivalent HPV vaccine to all boys and girls 11–12 years of age as well as to boys/men and girls/women 13–26 years of age who have not previously been vaccinated or who have not completed the full series. For women, Papanicolaou (Pap) smear testing and screening for HPV DNA are not recommended before vaccination. After vaccination, Pap testing is recommended to detect disease caused by other oncogenic HPV types. Because 30% of cervical cancers are caused by HPV types not contained in the vaccines, no changes in cervical cancer screening programs are currently recommended. Ongoing studies are examining the self testing for HPV to replace many Pap studies in patients with no evidence of cervical infection. Recent studies implicate HPV in some forms of squamous cell carcinoma of the oropharynx. The utility of current vaccines in preventing these cancers is not yet known.
++
++
++
A 32-year-old woman experiences an upper respiratory illness that began with rhinorrhea and nasal congestion. She also is complaining of a sore throat, but has no fever. Her illness lasts for about 5 days and resolves. Just prior to her illness, her 4-year-old child who attends daycare also experiences a similar illness. All of the following statements regarding the most common etiologic agent causing this illness are true EXCEPT:
++
++
++
A. Following the primary illness in a household, a secondary case of illness will occur is 25–70% of cases
++
++
B. The seasonal peak of the infection is in early fall and spring in temperate climates
++
++
C. The virus can be isolated from plastic surfaces up to 3 hours following exposure
++
++
D. The virus grows best at a temperature of 37°C, the temperature within the nasal passages
++
++
E. The virus is a single-stranded RNA virus of the Picornaviridae family
+
++
++
(Chap. 95) This patient presents with symptoms of the common cold with a self-limited illness characterized by rhinorrhea and sore throat. The most common viruses causing the common cold are rhinoviruses, implicated in as much as 50% of common colds. Rhinoviruses are small single-stranded RNA viruses of the Picornaviridae family. There are three genetic species of rhinoviruses with 102 serotypes identified. Rhinoviruses grow preferentially at the temperature of the nasal passages (33–34°C) rather than the temperature of the lower airways (37°C). While rhinovirus infections occur year round, there are seasonal peaks of the infection in the early fall and spring in temperate climates. Overall, the rates of rhinovirus infection are highest in infants and young children and decrease with age. The virus is most often introduced into families through young children in preschool or grade school. Following the index infection, secondary infections occur in other family members 25–70% of the time. Rhinovirus spreads through direct contact with infected secretions, which can occur through respiratory droplets or hand-to-hand contact. It can also be transmitted through large or small particle aerosols. Finally, virus can be isolated from plastic surfaces from 1 to 3 hours after inoculation, raising the possibility that virus can also be transmitted through environmental contact.
++
++
++
All of the following viruses are correctly matched with their primary clinical manifestation EXCEPT:
++
++
++
A. Adenovirus—Gingivostomatitis
++
++
B. Coronavirus—Severe acute respiratory syndrome
++
++
C. Human respiratory syncytial virus—Bronchiolitis in infants and young children
++
++
++
++
E. Rhinovirus—Common cold
+
++
++
(Chap. 95) The common viruses causing respiratory infections often have specific associated clinical syndromes. Rhinoviruses are primarily responsible for the common cold. Coronaviruses are also commonly associated with the cold. However, in 2002–2003, there was an outbreak of a coronavirus-associated illness that originated in China and spread to 28 countries in Asia, Europe, and North and South America. This illness was named severe acute respiratory syndrome (SARS) and caused severe lower respiratory illness and acute respiratory distress syndrome. Overall, the case-fatality rate was 9.5%. Human respiratory syncytial virus (HRSV) is the primary agent responsible for lower respiratory disease and bronchiolitis in infants and young children. Another virus primarily associated with childhood illness is parainfluenza virus. This virus is a frequent cause of croup in young children characterized by a febrile illness with a barking cough and stridor. Adenovirus often causes a febrile illness with common cold and pharyngitis in children. In adults, it is associated with outbreaks of respiratory illness in military recruits. Herpes simplex virus is associated gingivostomatitis in children and pharyngotonsillitis in adults.
++
++
++
In March 2009, the H1N1 strain of the influenza A virus emerged in Mexico and quickly spread worldwide over the next several months. Ultimately, over 18,000 people died due to the pandemic. This virus had genetic components of swine influenza viruses, an avian virus, and a human influenza virus. The genetic process by which this pandemic strain of influenza A emerged is an example of:
++
++
++
++
++
++
++
++
++
++
++
+
++
++
(Chap. 96) Pandemic strains of influenza emerge through genetic reassortment of RNA segments between viruses that affect different species, including humans, swine, and birds. This process is also called antigenic shift during which a new strain of influenza emerges to which very few people have immunity. Antigenic shift only occurs with influenza A as it is the only influenza that crosses between species. Antigenic drift is the result of point mutations in the hemagglutinin and/or neuraminidase proteins. Antigenic drift occurs frequently and is responsible for the interpandemic influenza outbreaks.
++
++
++
A 36-year-old man with HIV/AIDS (CD4+ lymphocyte count = 112/μL) develops a scaly, waxy, yellowish, patchy, crusty, pruritic rash on and around his nose. The rest of his skin examination is normal. Which of the following is the most likely diagnosis?
++
++
++
++
++
++
++
++
++
D. Reactivation herpes zoster
++
++
+
++
++
(Chap. 97) Dermatologic problems occur in >90% of patients with HIV infection. Seborrheic dermatitis is perhaps the most common rash in HIV patients, affecting up to 50% of patients. The prevalence increases with falling CD4+ T cell count. The rash involves the scalp and the face, appearing as described in the question. Therapy is standard topical treatment, although often a topical antifungal is added because of concomitant infection with Pityrosporum. Herpes zoster reactivation is painful and dermatomal, with progression of papules to vesicles to small pustules and then crusting. Molluscum contagiosum typically appears as one or many small pearly umbilicated asymptomatic papules occurring anywhere on the body. They can be a significant cosmetic issue in patients with AIDS. Psoriasis is not more common in patients with HIV infection but may be more severe and generalized. It would be uncommon to involve the face only. Kaposi’s sarcoma is due to coinfection with HHV-8 in patients with HIV/AIDS. It typically presents as >1 red/purple nodular painless lesions anywhere on the body.
++
++
++
Abacavir is a nucleoside transcription inhibitor that carries which side effect unique for HIV antiretroviral agents?
++
++
++
++
++
++
++
++
++
++
++
E. Severe hypersensitivity reaction
+
++
++
(Chap. 97) Abacavir use is associated with a potentially severe hypersensitivity reaction in about 5% of patients. There is likely a genetic component, with HLA-B*5701 being a significant risk factor for hypersensitivity syndrome. Symptoms, which usually occur within 2 weeks of therapy but can take >6 weeks to emerge, include fever, maculopapular rash, fatigue, malaise, gastrointestinal symptoms, and/or dyspnea. Once a diagnosis is suspected, the drug should be stopped and never given again because rechallenge can be fatal. For this reason, both the diagnosis and patient education once the diagnosis is made must be performed thoroughly and carefully. It is important to note that two available combination pills contain abacavir (epzicom, trizivir), so patients must know to avoid these as well. Fanconi’s anemia is a rare disorder associated with tenofovir. Zidovudine causes anemia and sometimes granulocytopenia. Stavudine and other nucleoside reverse transcriptase inhibitors are associated with lipoatrophy of the face and legs.
++
++
++
A 38-year-old man with HIV/AIDS presents with 4 weeks of diarrhea, fever, and weight loss. Which of the following tests makes the diagnosis of cytomegalovirus (CMV) colitis?
++
++
++
++
++
B. Colonoscopy with biopsy
++
++
C. Serum CMV polymerase chain reaction (PCR)
++
++
++
++
+
++
++
(Chap. 97) CMV colitis should be considered in AIDS patients with CD4+ lymphocyte count <50/μL, fevers, and diarrhea. Diarrhea is often bloody but can be watery. Initial evaluation often involves stool studies to rule out other parasitic or bacterial causes of diarrhea in AIDS patients. A standard panel will include some or all of the following depending on epidemiologic and historical data: C. difficile stool antigen, stool culture, stool Mycobacterium avium intracellulare culture, stool ova and parasite examination and special stains for Cryptosporidium, Isospora, Cyclospora, and Microsporidium. There is no stool or serum test that is useful for the evaluation of CMV colitis in an HIV-infected patient. A positive CMV IgG is merely a marker of past infection. If this test is negative, then the pretest probability of developing active CMV decreases substantially. Serum CMV PCR has gained utility in solid organ and bone marrow transplant patients for following treatment response for invasive CMV infection. However, in HIV-infected patients, CMV viremia correlates imprecisely with colitis. Further, because CMV is a latent-lytic herpesvirus, a positive serum PCR does not imply disease unless drawn in the right clinical context, for which there is none in HIV infection. Colonic histology is sensitive and specific for the diagnosis of CMV colitis, with large-cell inclusion bodies being diagnostic.
++
++
++
In the scenario described above, what would be the best approach to prevent development of chronic hepatitis?
++
++
++
A. Administration of anti-hepatitis A virus IgG
++
++
B. Administration of lamivudine
++
++
C. Administration of pegylated interferon alfa plus ribavirin
++
++
D. Administration of prednisone beginning at a dose of 1 mg/kg daily
++
++
E. Do nothing and observe as 99% of individuals with this disease recover
+
++
++
(Chap. 99) No treatment is recommended for acute hepatitis B in most individuals because 99% of infected individual recover without assistance. Therefore, it would not be expected that an individual would derive any particular benefit from treatment. In severe acute hepatitis B, nucleoside analogues, including lamivudine, have been used successfully although there are no clinical trial data to support such an approach. Hepatitis A is an acute and self-limited illness that does not progress to chronic liver disease. Thus, no treatment is required. Anti-hepatitis A virus immunoglobulin can be give prophylactically following a known exposure to prevent development of disease, but it is not helpful in established disease. There is no role for oral or intravenous corticosteroids in the treatment of acute viral hepatitis of any etiology. It has demonstrated no clinical benefit and may increase the risk of developing chronic disease.
++
++
++
In chronic hepatitis B virus (HBV) infection, presence of hepatitis B e antigen (HBeAg) signifies which of the following?
++
++
++
A. Development of liver fibrosis leading to cirrhosis
++
++
B. Dominant viral population is less virulent and less transmissible
++
++
C. Increased likelihood of an acute flare in the next 1–2 weeks
++
++
D. Ongoing viral replication
++
++
+
++
++
(Chaps. 99 and 100) In the course of acute hepatitis B, HBeAg positivity is common and usually transient. Persistence of HBeAg in the serum for >3 months indicates an increased likelihood of development of chronic hepatitis B. In chronic hepatitis B, presence of HBeAg in the serum indicates ongoing viral replication and increased infectivity. It is also a surrogate for inflammatory liver injury but not fibrosis. The development of antibody to HBeAg (anti-HBe) is indicative of the nonreplicative phase of HBV infection. During this phase, intact virions do not circulate and infectivity is less. Currently, quantification of HBV DNA with polymerase chain reaction allows risk stratification as <103 virions/µL is the approximate threshold for liver injury and infectivity.
++
++
++
A 46-year-old man is known to have chronic hepatitis C virus (HCV) infection. He is a former intravenous drug user for more than 20 years who has been abstinent from drug use for 1 year. He was treated for tricuspid valve endocarditis 3 years previously. He does not know when he acquired HCV. His laboratory studies show a positive HCV IgG antibody with a viral load of greater than 1 million copies. The virus is genotype 2. His AST is 82 U/L, and his ALT is 74 U/L. He undergoes liver biopsy which demonstrates a moderate degree of bridging fibrosis. Which of the following is the most predictive of the development of cirrhosis?
++
++
++
A. Abnormal transaminases
++
++
B. Bridging fibrosis on liver biopsy
++
++
++
++
D. History of bacterial endocarditis
++
++
E. History of IV drug use
+
++
++
(Chap. 100) Chronic hepatitis develops in about 85% of all individuals affected with HCV, and 20–25% of these individuals will progress to cirrhosis over about 20 years. Among those infected with HCV, about one-third of individuals will have normal or near-normal levels of aminotransferases although liver biopsy demonstrates active hepatitis as much as one-half of patients. Moreover, about 25% of individuals with normal aminotransferase levels at one point in time will develop elevations in these enzymes later and can lead to progressive liver disease. Thus, normal aminotransferase levels at a single point in time do not definitively rule out the possibility that cirrhosis can develop. Progression to end stage liver disease in individuals with chronic HCV hepatitis is more likely in older individuals, longer duration of infection, advanced histologic stage and grade, genotype 1 infection, more complex quasispecies diversity, concomitant other liver disease, HIV infection, and obesity. Among these factors, the best prognostic indicator for the development of progressive liver disease is liver histology. Specifically, patients who have moderate to severe inflammation or necrosis including septal or bridging fibrosis have the greatest risk of developing cirrhosis over the course of 10–20 years.
++
++
++
A 28-year-old woman is evaluated for fever, anorexia and malaise. She has a 4-year-old son who recently has had a similar illness and now has the findings shown in Figure 89.
++
++
++
++
(Images reprinted courtesy of Centers for Disease Control and Prevention/Emerging Infectious Diseases.)
++
++
What is the most likely etiologic agent for this patient’s presentation?
++
++
++
++
++
++
++
++
++
D. Herpes simplex virus 1
++
++
+
++
++
(Chap. 101) The image demonstrates a child with vesicular eruptions on the hand, foot and mouth. This constellation of findings is characteristic of a diagnosis of hand-foot-and-mouth disease, a viral infection caused by coxsackievirus A16 or enterovirus 71. The infection has an incubation period of 4–6 days and then begins with symptoms of fever, malaise, and anorexia. Following this, vesicles appear on the buccal mucosa, tongue, and the dorsum of the hand. Only about one-third of affected individuals have a rash on the feet. The lesions are tender but are non-pruritic. Lesions may also appear on the palate, uvula, or tonsillar pillars. The infection is highly contagious with nearly 100% attack rates among exposed children. However, the disease is self-limited with recovery within 1 week without consequence.
++
++
++
You are treating a 5-year-old child who was not vaccinated for measles. The child was exposed at a playground where an individual who had the disease was present. The child is having fevers to 102.1°F, has a diffuse erythematous macular rash, and Koplik’s spots. What treatment do you recommend at this time?
++
++
++
A. Intravenous immunoglobulin
++
++
++
++
C. Prophylactic antibacterial therapy with a penicillin or cephalosporin to prevent pneumonia
++
++
++
++
+
++
++
(Chap. 102) There is no specific antiviral therapy for measles. Treatment should include supportive measures such as hydration and management of fever. If a secondary bacterial infection is suspected based upon clinical findings, such as pneumonia or otitis media, then antibiotics should be initiated at that time. There is no evidence to support prophylactic use of antibiotics. Vitamin A is effective for the treatment of measles and can reduce morbidity and mortality rates. The World Health Organization recommends administering vitamin A 200,000 units orally once per day for two consecutive days. A third dose is recommended 2–4 weeks later if there is documented vitamin A deficiency.
++
++
++
A 23-year-old previously healthy female letter carrier works in a suburb in which the presence of rabid foxes and skunks has been documented. She is bitten by a bat, which then flies away. Initial examination reveals a clean break in the skin in the right upper forearm. She has no history of receiving treatment for rabies and is unsure about vaccination against tetanus. The physician should
++
++
++
A. clean the wound with a 20% soap solution
++
++
B. clean the wound with a 20% soap solution and administer tetanus toxoid
++
++
C. clean the wound with a 20% soap solution, administer tetanus toxoid, and administer human rabies immune globulin intramuscularly
++
++
D. clean the wound with a 20% soap solution, administer tetanus toxoid, administer human rabies immune globulin IM, and administer human diploid cell vaccine
++
++
E. clean the wound with a 20% soap solution and administer human diploid cell vaccine
+
++
++
(Chap. 105) The patient has been bitten by a member of a species known to carry rabies in an area in which rabies is endemic. Based on the animal vector and the facts that the skin was broken and that saliva possibly containing the rabies virus was present, postexposure rabies prophylaxis should be administered. If an animal involved in an unprovoked bite can be captured, it should be killed humanely and the head should be sent immediately to an appropriate laboratory for rabies examination by the technique of fluorescent antibody staining for viral antigen. If a healthy dog or cat bites a person in an endemic area, the animal should be captured, confined, and observed for 10 days. If the animal remains healthy for this period, the bite is highly unlikely to have transmitted rabies. Postexposure prophylactic therapy includes vigorous cleaning of the wound with a 20% soap solution to remove any virus particles that may be present. Tetanus toxoid and antibiotics should also be administered. Passive immunization with anti-rabies antiserum in the form of human rabies immune globulin (rather than the corresponding equine antiserum because of the risk of serum sickness) is indicated at a dose of 10 units/kg into the wound and 10 units/kg IM into the gluteal region. Second, one should actively immunize with an anti-rabies vaccine [either human diploid cell vaccine or rabies vaccine absorbed (RVA)] in five 1-mL doses given IM, preferably in the deltoid or anterior lateral thigh area. The five doses are given over a 28-day period. The administration of either passive or active immunization without the other modality results in a higher failure rate than does the combination therapy.
++
++
++
While working at a new medical school in Kuala Lumpur, Malaysia, a 40-year-old previously healthy male from Baltimore develops sudden onset malaise, fever, headache, retro-orbital pain, backache and myalgias. On examination his temperature is 39.6°C with normal blood pressure and slight tachycardia. He has some vesicular lesions on his palate and scleral injection. Laboratory studies are notable for a platelet count of 80,000/ul. All of the following are true regarding his illness EXCEPT:
++
++
++
A. A second infection could result in hemorrhagic fever
++
++
B. After resolution, he has lifelong immunity
++
++
C. IgM ELISA may be diagnostic
++
++
D. In equatorial areas, year round transmission occurs
++
++
E. The disease is transmitted by mosquito
+
++
++
(Chap. 106) This patient has a typical presentation of Dengue fever. All four distinct dengue viruses (dengue 1–4) have the mosquito Aedes aegypti as their principal vector, and all cause a similar clinical syndrome. Thus, lifelong immunity cannot be presumed. In rare cases, second infection with a serotype of dengue virus different from that involved in the primary infection leads to dengue hemorrhagic fever with severe shock. Year-round transmission between latitudes 25°N and 25°S has been established and seasonal forays of the viruses to points as far north as Philadelphia are thought to have taken place in the United States. Dengue fever is seen in throughout SE Asia including Malaysia, Thailand, Viet Nam, and Singapore. In the Western Hemisphere, it may be found in the Caribbean region, including Puerto Rico. With increasing spread of the vector mosquito throughout the tropics and subtropics, large areas of the world have become vulnerable to the introduction of dengue viruses, particularly through air travel by infected humans, and both dengue fever and the related dengue HF are becoming increasingly common. The A. aegypti mosquito, which is also an efficient vector of the yellow fever and chikungunya viruses, typically breeds near human habitation, using relatively freshwater from sources such as water jars, vases, discarded containers, coconut husks, and old tires. A. aegypti usually inhabits dwellings and bites during the day. After an incubation period of 2–7 days, the typical patient experiences the symptoms described above along with the severe myalgia that gave rise to the colloquial designation “break-bone fever.” There is often a macular rash on the first day as well as adenopathy, palatal vesicles, and scleral injection. The illness may last a week, with additional symptoms usually including anorexia, nausea or vomiting, marked cutaneous hypersensitivity, and—near the time of defervescence—a maculopapular rash beginning on the trunk and spreading to the extremities and the face. Laboratory findings include leukopenia, thrombocytopenia, and, in many cases, serum aminotransferase elevations. The diagnosis is made by IgM ELISA or paired serology during recovery or by antigen-detection ELISA or RT-PCR during the acute phase. In endemic regions where specific testing is not easily available the diagnosis is presumed in cases of a typical clinical presentation and thrombocytopenia. Given the frequency of disease and the potential for hemorrhagic fever, active investigation is pursuing an effective vaccine.
++
++
++
All of the following statements regarding Ebola virus disease (EVD) are true EXCEPT:
++
++
++
A. Abdominal pain and diarrhea are common manifestations of EVD
++
++
B. EVD is frequently spread human-to-human via respiratory aerosols
++
++
C. Patients who survive EVD often have severe sequelae such as arthralgias and asthenia
++
++
D. Treatment of EVD is entirely supportive
++
++
E. Viruses of the Filoviridae family cause EVD
+
++
++
(Chap. 107) Several viruses of the family Filoviridae cause severe and frequently fatal viral hemorrhagic fevers in humans. The family Filoviridae includes three genera: Cuevavirus, Ebolavirus, and Marburgvirus. The available data suggest that the only known cuevavirus, Lloviu virus (LLOV), and one ebolavirus, Reston virus (RESTV), are not pathogenic for humans. The remaining four ebolaviruses—Bundibugyo virus (BDBV), Ebola virus (EBOV), Sudan virus (SUDV), and Taï Forest virus (TAFV)—cause Ebola virus disease (EVD).
++
Introduction of filoviruses into human populations is an extremely rare event that most likely occurs by direct or indirect contact with healthy mammalian filovirus hosts or by contact with infected, sick, or deceased nonhuman primates. Filoviruses are highly infectious but not very contagious. Natural human-to-human transmission takes place through direct person-to-person (usually skin-to-skin) contact or exposure to infected bodily fluids and tissues; there is no evidence of such transmission by aerosol or respiratory droplets. The first phase (disease onset until around day 5–7) resembles influenza and is characterized by sudden onset of fever and chills, severe headaches, cough, myalgia, pharyngitis, arthralgia of the larger joints, development of a maculopapular rash, and other systemic signs/symptoms. The second phase (approximately 5–7 days after disease onset and thereafter) typically involves the gastrointestinal tract (abdominal pain with vomiting and/ or diarrhea) respiratory tract (chest pain, cough), vascular system (postural hypotension, edema), and central nervous system (confusion, coma, headache). Hemorrhagic manifestations such as subconjunctival injection, nosebleeds, hematemesis, hematuria, and melena are also typical. Common laboratory findings are leukopenia (with cell counts as low as 1000/μL) with a left shift prior to leukocytosis, thrombocytopenia (with counts as low as 50,000/μL), increased concentrations of liver and pancreatic enzymes, hypokalemia, hypoproteinemia, increased creatinine and urea concentrations with proteinuria, and prolonged prothrombin and partial thromboplastin times. Patients usually succumb to disease 4–14 days after infection. Patients who survive experience prolonged and sometimes incapacitating sequelae such as arthralgia, asthenia, iridocyclitis, hearing loss, myalgia, orchitis, parotitis, psychosis, recurrent hepatitis, transverse myelitis, or uveitis. Temporary hair loss and desquamation of skin areas previously affected by a typical maculopapular rash are visible consequences of the disease. Rarely, filoviruses can persist in the liver, eyes, or testicles of survivors and may cause recurrent disease months after convalescence. Any treatment of patients with suspected or confirmed filovirus infection must be administered under increased safety precautions by experienced specialists using appropriate personal protective equipment. Treatment of EVD is entirely supportive because no accepted/approved, efficacious, specific antiviral agents or vaccines are yet available.
++
++
++
A 24-year-old female student at the Ohio State University is seen in the emergency department for shortness of breath and chest pain. She has no significant past medical history and grew up Cincinnati. Her only medication is an oral contraceptive. As a component of her evaluation she receives a contrast enhanced CT scan of the chest. Fortunately, there is no pulmonary embolism (she is diagnosed with viral pleuritis) but there are numerous lung, mediastinal, and splenic calcifications. Based on these findings, which of the following remote infections was most likely?
++
++
++
++
++
++
++
++
++
++
++
+
++
++
(Chap. 111) All of these pathogens are typically inhaled and cause pulmonary infection, which may resolve spontaneously or progress to active disease. Resolved infection with blastomycosis, coccidioidomycosis, Cryptococcus, and tuberculosis will often leave a radiographic lesion that typically looks like a solitary nodule and may be confused with potential malignancy. Latent tuberculosis is often suggested by the radiographic finding of a calcified lymph node that is typically solitary. Of the listed infections, histoplasmosis is most likely to resolve spontaneously in an immunocompetent individual leaving multiple mediastinal and splenic calcifications. These represent calcified granulomas formed after an appropriate cellular immunity response involving interleukin 12, TNF-a in combination with functional lymphocytes, macrophages, and epithelial cells. In endemic areas, such as the Ohio and Mississippi river valleys in the United States, 50–80% of adults have evidence of previous infection without clinical manifestations. In patients with impaired cellular immunity, the infection may disseminate to the bone marrow, spleen, liver, adrenal glands, and mucocutaneous membranes. Unlike tuberculosis, remote histoplasma infection rarely reactivates.
++
++
++
A 62-year-old man returns from a vacation to Arizona with fever, pleurisy, and a nonproductive cough. All of the following factors on history and laboratory examination favor a diagnosis of pulmonary coccidioidomycosis rather than community-acquired pneumonia EXCEPT:
++
++
++
++
++
++
++
C. Mediastinal lymphadenopathy on chest roentgenogram
++
++
D. Positive Coccidioides complement fixation titer
++
++
E. Travel limited to Northern Arizona (Grand Canyon area)
+
++
++
(Chap. 112) Northern Arizona (i.e., the Grand Canyon region) is not a region of high incidence of coccidioidomycosis. Coccidioidomycosis is confined to the Western Hemisphere between the latitudes of 40°N and 40°S. In the United States, areas of high endemicity include the southern portion of the San Joaquin Valley of California and the south-central region of Arizona. However, infection may be acquired in other areas of the southwestern United States, including the southern coastal counties in California, southern Nevada, southwestern Utah, southern New Mexico, and western Texas, including the Rio Grande Valley. Outside the United States, coccidioidomycosis is endemic to northern Mexico as well as to localized regions of Central America. In South America, there are endemic foci in Colombia, Venezuela, northeastern Brazil, Paraguay, Bolivia, and north-central Argentina. Eosinophilia is a common laboratory finding in acute coccidioidomycosis and erythema nodosum is a common cutaneous clinical feature (particularly on the lower extremities in women). Mediastinal lymphadenopathy is more commonly seen on radiographs for all acute pneumonias due to endemic mycoses, including Coccidioides, rather than due to bacterial pneumonia. A positive complement fixation test is one method to definitively diagnose acute infection.
++
++
++
In a patient with lung and skin lesions a travel history to which of the following regions would be most compatible with the potential diagnosis of Blastomycosis?
++
++
++
A. Brazil (Amazon river basin)
++
++
++
++
++
++
++
++
E. Western Washington state
+
++
++
(Chap. 113) Blastomycosis is caused by the dimorphic fungus, B. dermatitides, which commonly resides in soil and is acquired through inhalation. Pulmonary infection is most common and can be acute or indolent. Extrapulmonary extension via hematogenous spread from the lungs is common with skin lesions and osteomyelitis most common. In patients with AIDS, CNS involvement, usually as a brain abscess, has been reported in approximately 40% of cases of blastomycosis. Most cases of blastomycosis are reported from North America with the most common regions being bordering the Mississippi and Ohio River basins, the upper Midwest and Canada bordering the Great Lakes, and a small area of New York and Ontario bordering the St. Lawrence river. Outside of North American most Blastomycosis cases are in Africa. Coccidioidomycosis is endemic in Southern Arizona.
++
++
++
A 34-year-old female aviary worker who has no significant past medical history, is taking no medications, has no allergies, and is HIV-negative presents to the emergency room with fever, headache, and fatigue. She reports that her headache has been present for at least 2 weeks, is bilateral and worsened by bright lights and loud noises. She is typically an active person who has recently been fatigued and has lost 8 lbs due to anorexia. Her work involves caring for birds and maintaining their habitat. Her vital signs are notable for a temperature of 101.8°F. Neurologic examination is normal except for notable photophobia. Head CT examination is normal. Lumbar puncture is significant for an opening pressure of 20 cm H2O, white blood cell count of 15 cells/μL (90% monocytes), protein of 0.5 g/L (50 mg/mL), glucose of 2.8 mmol/L (50 mg/dL), and positive India ink stain. What is the appropriate therapy for this patient?
++
++
++
A. Amphotericin B for 2 weeks followed by lifelong fluconazole
++
++
B. Amphotericin B plus flucytosine for 2 weeks followed by oral fluconazole for 10 weeks
++
++
C. Caspofungin for 3 months
++
++
D. Ceftriaxone and vancomycin for 2 weeks
++
++
E. Voriconazole for 3 months
+
++
++
(Chap. 114) The goal of therapy for cryptococcal meningoencephalitis in an HIV-negative patient is cure of the fungal infection, not simply control of symptoms. Therefore, lifelong therapy is not generally necessary. Pulmonary cryptococcosis in an immunocompetent host sometimes resolves without therapy. However, given the propensity of Cryptococcus species to disseminate from the lung, the inability to gauge the host’s immune status precisely, and the availability of low-toxicity therapy in the form of fluconazole, the current recommendation is for pulmonary cryptococcosis in an immunocompetent individual to be treated with fluconazole (200–400 mg/d for 3–6 months). Extrapulmonary cryptococcosis without CNS involvement in an immunocompetent host can be treated with the same regimen, although amphotericin B (AmB; 0.5–1 mg/kg daily for 4–6 weeks) may be required for more severe cases. For CNS involvement in a host without AIDS or obvious immune impairment, most authorities recommend initial therapy with AmB (0.5–1 mg/kg daily) during an induction phase, which is followed by prolonged therapy with fluconazole (400 mg/d) during a consolidation phase. For cryptococcal meningoencephalitis without a concomitant immunosuppressive condition, the recommended regimen is AmB (0.5–1 mg/kg) plus flucytosine (100 mg/kg) daily for 6–10 weeks. Alternatively, patients can be treated with AmB (0.5–1 mg/kg) plus flucytosine (100 mg/kg) daily for 2 weeks and then with fluconazole (400 mg/d) for at least 10 weeks. Patients with immunosuppression are treated with the same initial regimens except that consolidation therapy with fluconazole is given for a prolonged period to prevent relapse. Neither caspofungin nor micafungin have activity against Cryptococcus. Voriconazole and posaconazole are highly active against cryptococcal strains and appear effective clinically, but clinical experience with these agents in the treatment of cryptococcosis is limited. Ceftriaxone and vancomycin are the recommended treatments for bacterial meningitis in an immunocompetent patient <50 years of age and have no role in the therapy of Cryptococcus.
++
++
++
Which of the following statements regarding the use of antifungal agents to prevent candida infections is true?
++
++
++
A. HIV-infected patients should receive prophylaxis for oropharyngeal candidiasis when CD4 count is <200
++
++
B. Most centers administer fluconazole to recipients of allogeneic stem cell transplants
++
++
C. Most centers administer fluconazole to recipients of living related renal transplants
++
++
D. Voriconazole has been shown to be superior to other agents as prophylaxis in liver transplant recipients
++
++
E. Widespread candida prophylaxis in postoperative patients in the SICU has been shown to be cost effective
+
++
++
(Chap. 115) The use of antifungal agents to prevent Candida infections remains controversial but some general principles have emerged in recent years. Most centers start prophylactic fluconazole to allogeneic stem cell transplant recipients. Many centers also administer them to high-risk liver transplant recipients, but not routine living related renal transplant recipients. This prophylaxis should be differentiated from the administration of empiric broad-spectrum antifungal therapy in a patient with prolonged febrile neutropenia. Voriconazole is an appropriate choice for empiric broad spectrum therapy in an unstable patient with suspected candidemia, but it has not been shown to be superior to any other agent for prophylaxis against candida in any population. Complicated postoperative surgical patients are at risk of candida infection and some centers administer prophylaxis to very high risk patients. However, the widespread use of candida prophylaxis in surgical patients is not recommended because the incidence of disseminated candidiasis is low, the cost-benefit ratio is suboptimal, and there is reasonable rationale to believe that this strategy could increase candida resistance to current medications. Candida prophylaxis for HIV-infected patients is recommended to prevent frequent recurrent oropharyngeal or esophageal infection.
++
++
++
A local oncology center is concerned about the occurrence of an outbreak of cases of invasive aspergillus in patients receiving bone marrow transplants. Which of the following is the most likely source of aspergillus infection?
++
++
++
A. Contaminated air source
++
++
B. Contaminated water source
++
++
C. Patient-to-patient spread in outpatient clinic waiting rooms
++
++
D. Provider-to-patient spread due to poor hand washing technique
++
++
E. Provider-to-patient spread due to poor utilization of alcohol disinfectant
+
++
++
(Chap. 116) Aspergillus has a worldwide distribution, typically growing in decomposing plant materials. Immunocompetent individuals generally do not develop disease without intense exposure such as during construction or handling of moldy hay, bark, or compost. Nosocomial outbreaks are usually directly related to contaminated air source in the hospital. HEPA filtration is effective in eliminating infection from ORs and units with high risk patients. Contaminated water sources are the typical reservoir of nosocomial Legionella outbreaks. Patient-to-patient spread in waiting rooms has been described for cystic fibrosis patients transmitting Burkholderia infection. Provider-to-patient transmission of MRSA and most other bacteria is reduced with effective use of alcohol-based disinfectant; however, in the case of C. difficile, alcohol will not eliminate spores and effective hand washing with soap/water is necessary.
++
++
++
Which of the following is the most common form of infection in patients with mucormycosis?
++
++
++
++
++
++
++
C. Hematogenous dissemination
++
++
++
++
+
++
++
(Chap. 117) The sites of infection due to Mucorales fungal infection tend to affect patients with specific host defense defects. The most common clinical manifestation of mucormycosis is rhinocerebral. Most cases occur in patients with diabetes or hyperglycemia due to glucocorticoid therapy (e.g., solid organ transplantation). The initial symptoms usually include facial/orbital pain or numbness, facial suffusion and soft tissue swelling. The infection usually originates in the ethmoid sinus region and will spread rapidly to the orbit and CNS. Painful necrotic lesions may be seen in the mouth. Pulmonary mucormycosis is the second most common manifestation of Mucorales infection. Human stem cell transplantation is a common risk factor for pulmonary mucormycosis. The risk factors and presentation are similar to invasive pulmonary aspergillus. Differentiation is important as antifungal therapy differs. The two diseases appear similar on chest CT although the presence of >10 nodules, pleural effusion or concomitant sinusitis makes mucormycosis more likely. Other sites of involvement with mucormycosis are described but less common. Cutaneous disease may result from external implantation (soil related trauma or plant penetration) or hematogenous dissemination. Implanted cutaneous disease is also highly invasive; the development of fasciitis has a >70% mortality. Rapid surgical debridement is essential. Hematogenous dissemination has a very high mortality; involvement of the brain has a near 100% mortality. Gastrointestinal mucormycosis is most common in neonates with necrotizing enterocolitis.
++
++
++
A 21-year-old college student seeks your opinion because of a lesion on his head. He has no significant medical history and reports a solitary lesion on the crown of his head for over 1 month that has been growing slowly. He has had no fever and reports that while the area is itchy he feels well. On examination you note a 3-cm round area of alopecia without redness, pain, or inflammation. It is well demarcated with central clearing, scaling, and broken hair shafts at the edges. There is no redness or pain. Which of the following should you recommend?
++
++
++
++
++
++
++
++
++
++
++
+
++
++
(Chap. 118) This patient has tinea capitis most likely caused by the dermatophytic mold, Trichophyton. The other dermatophytes that less frequently cause cutaneous infection include Microsporum and Epidermophyton. They are not part of the normal skin flora but can live in keratinized skin structures. Infections with these organisms are extremely common and are often called ringworm, although the causative organisms are fungi not worms. They manifest as infection of the head (tinea capitis), feet (tinea pedis), crotch (tinea cruris), and nails (tinea unguium or onychomycosis). Tinea capitis is most common in children aged 3–7 but also occurs in adults. Usually the typical appearance, as in this case, is diagnostic. Scrapings may be taken from the edge of lesion and stained with KOH to reveal hyphae. Dermatophyte infections often respond to topical therapy. For troublesome infections, itraconazole or terbinafine for 1–2 weeks can hasten resolutions. Terbinafine is often preferred because of fewer drug interactions.
++
++